Chapter 6. Types and Pain Flashcards

1
Q
  1. A45-year-old patient with metastatic breast carcinoma
    is prescribed 30 mg of sustained-release
    morphine (MS Contin) twice a day and one
    15-mg tablet of immediate-release morphine
    (MSIR) every 6 hours as needed for breakthrough
    pain. On her routine follow-up visit she
    reports that she routinely uses MSIR four times
    a day with satisfactory pain control on most days
    and no major side effects. What would be your
    best course of action in this situation?
    (A) Prescriptions should be left unchanged
    (B) MS Contin should be changed to 40 mg
    of OxyContin twice a day and 5 mg of
    oxycodone every 6 hours as needed for
    breakthrough pain
    (C) Fentanyl patch of 25 μg/h should
    replace MS Contin with 15 mg of MSIR
    every 6 hours as needed for breakthrough
    pain
    (D) MS Contin should be increased to 60 mg
    twice a day with MSIR 15 mg every
    6 hours as needed for breakthrough pain
    (E) MS Contin should be increased to
    60 mg twice a day, and MSIR should be
    discontinued
A
  1. (D) If a patient routinely uses breakthrough
    medications, the daily total amount should be
    converted to a sustained-release dose and
    added to the current maintenance dose.
How well did you know this?
1
Not at all
2
3
4
5
Perfectly
2
Q
333. Approximately in what percentage of patients
with malignancies does pain unrelated to cancer
occur?
(A) Less than 2%
(B) 3%
(C) 7.5%
(D) 11%
(E) 25%
A
  1. (B) Approximately 3% of pain syndromes in
    cancer patients are unrelated to the underlying
    malignancy or cancer treatment. Most commonly,
    pain is caused by degenerative disc disease,
    arthritis, fibromyalgia, or migraine and
    has often predated the diagnosis of cancer.
How well did you know this?
1
Not at all
2
3
4
5
Perfectly
3
Q
  1. There is a significant incidence of neuropathic
    pain in a cancer patient with brachial plexopathy.
    The etiology of the brachial plexopathy in such
    a patient may be caused by direct tumor infiltration
    or radiation fibrosis. Electrophysiologic
    evaluation with nerve conduction velocity (NCV)
    study and electromyography (EMG) helps to
    distinguish between the two etiologies. Which of
    the following findings of NCV/EMG is the most
    helpful to differentiate between the direct tumor
    infiltration and the radiation fibrosis etiologies of
    brachial plexopathy?
    (A) Segmental nerve conduction slowing
    (B) Myokymia
    (C) Fibrillation potentials
    (D) Positive sharp waves
    (E) Decreased amplitude of the compound
    muscle action potential (CMAP)
A
  1. (B) Segmental nerve conduction slowing, fibrillation
    potentials, positive sharp waves, and
    decreased amplitude CMAPs are all helpful in
    determining the presence of brachial plexopathy
    in general. Myokymia is present in 63% of
    patients with radiation fibrosis induced
    brachial plexopathy. Brachial plexopathy
    caused by direct tumor infiltration has a low
    incidence of myokymia. Myokymia is a continuous
    but brief involuntary muscle twitching
    that gives the appearance of wormlike rippling
    of the muscle. It
How well did you know this?
1
Not at all
2
3
4
5
Perfectly
4
Q
335. If bony metastases are present, which primary
cancer location has the best 5-year survival
prognosis?
(A) Myeloma
(B) Breast
(C) Prostate
(D) Thyroid
(E) Kidney
A
  1. (D) A5-year survival for a cancer patient with
    documented skeletal metastases varies widely
    depending on location of the primary tumor:
    myeloma—10%; breast—20%; prostate—25%;
    lung—less than 5%; kidney—10%; thyroid—
    40%; melanoma—less than 5%.
How well did you know this?
1
Not at all
2
3
4
5
Perfectly
5
Q
  1. The most frequent spinal cord symptom or sign
    in patients with carcinomatous meningitis is
    (A) nuchal rigidity
    (B) back pain
    (C) reflex asymmetry
    (D) positive straight leg raise test
    (E) weakness
A
  1. (C) Reflex asymmetry occurs in 67% of patients
    with carcinomatous meningitis and is the most
    frequent spinal cord–related sign. The frequency
    of nuchal rigidity, back pain, positive straight leg raise test, and weakness is 11%,
    25%, 13%, and 33%, respectively.
How well did you know this?
1
Not at all
2
3
4
5
Perfectly
6
Q
  1. Which of the following would most likely be
    responsible for the central pain syndrome?
    (A) Epidural spinal cord compression
    (B) Metastatic bony destruction of the
    vertebrae with a nerve root compression
    (C) Metastatic involvement of the cranial
    nerves
    (D) Carcinomatous meningitis
    (E) Radiation myelopathy
A
  1. (E) Central pain syndromes are relatively rare in
    cancer patients. Although epidural spinal cord
    compression is almost always painful, central pain
    is not the predominant symptom. Nociceptive
    input from progressive bony destruction by
    metastases is the usual cause of pain, with or without
    concurrent radicular pain from nerve root
    compression. Radiation myelopathy is the central
    pain syndrome.
How well did you know this?
1
Not at all
2
3
4
5
Perfectly
7
Q
338. The majority of patients with epidural metastasis
have the following pattern of pain:
(A) Local
(B) Radicular
(C) Referred
(D) Funicular
(E) All of the above
A
  1. (A) The most common pattern of pain in patients
    with epidural metastasis is local. Local pain over
    the involved vertebral body, which results from
    the involvement of the vertebral periosteum, is
    dull and exacerbated by recumbency.
    Radicular pain from compressed or damaged
    nerve roots is usually unilateral in the
    cervical and lumbosacral regions and bilateral
    in the thorax. The pain is experienced in the
    overlying spine, deep in certain muscles supplied
    by the compressed root, and in the cutaneous
    distribution of the injured root.
    Referred pain has a deep aching quality
    and is often associated with tenderness of
    subcutaneous tissues and muscles at the site
    of referral. The typical examples of referred
    pain pattern include buttocks and posterior
    thigh pain with lumbosacral spine involvement;
    pain in the flank, groin, and anterior
    thigh in the upper lumbar spine involvement;
    midscapular and shoulder pain in the cervicothoracic
    epidural disease.
    Funicular pain usually occurs some distance
    below the site of compression and it has hot or cold qualities in a poorly localized nondermatomal
    distribution. It presumably results
    from compression of the ascending sensory
    tracts in the spinal cord.
How well did you know this?
1
Not at all
2
3
4
5
Perfectly
8
Q
  1. All of the following are true about the World
    Health Organization (WHO) analgesic ladder,
    EXCEPT
    (A) it is a method for relief of cancer pain
    based on a small number of relatively
    inexpensive drugs
    (B) it has three steps
    (C) step one involves the use of opioids
    (D) it suggests to use only one drug from
    each group at a time
    (E) it is a simple and effective method for
    controlling cancer pain
A
  1. (C) The WHO analgesic ladder is based on the
    premise that most patients throughout the world
    gain adequate pain relief if health care professionals
    learn how to use a few effective and
    relatively inexpensive drugs well. Step 1 of the
    ladder involves the use of nonopioids. If this
    step is ineffective, go to step 2 and add an
    opioid for mild to moderate pain. Step 3 substitutes
    an opioid for moderate to severe pain
    in step 2. Only one drug from each group
    should be used at a time. Adjuvant drugs can
    be used in all steps.
How well did you know this?
1
Not at all
2
3
4
5
Perfectly
9
Q
340. The following are all true about methadone,
EXCEPT
(A) it has a highly variable oral bioavailability
(B) it is a low cost medication
(C) it has no known active metabolites
(D) it has N-methyl-D-aspartate (NMDA)
receptor agonist properties
(E) it has high lipid solubility
A
  1. (D) Methadone has a variable oral bioavailability
    between 41% and 99% and, therefore,
    should be started with extra caution (low initial
    dose and slow subsequent increases).
    Methadone differs from all other opioids by its
    noncompetitive antagonist activity at the
    NMDAreceptors. Activation of NMDA receptors
    has been shown to play a role in development
    of tolerance to analgesic effects of
    opioids, as well as in the pathologic sensory
    states, such as neuropathic pain, inflammatory
    pain, ischemic pain, allodynia, and spinal
    states of hypersensitivity.
How well did you know this?
1
Not at all
2
3
4
5
Perfectly
10
Q
  1. 58-years-old patient with metastatic prostate
    cancer is taking sustained-release morphine
    (MS Contin) every 8 hours with a total daily
    dose of 225 mg with optimal pain control.
    Because of some circumstances, he has to be
    converted to transdermal therapeutic system
    fentanyl (TTS-fentanyl). What is the correct
    dose of fentanyl patch equivalent to the current
    dose of MS Contin for this patient?
    (A) 25 μg/h every 72 hours
    (B) 50 μg/h every 48 hours
    (C) 75 μg/h every 72 hours
    (D) 100 μg/h every 48 hours
    (E) 125 μg/h every 72 hours
A
  1. (C) As a rough guide for conversion, the 8-hourly
    dose of MS Contin (225/3 = 75 mg in this case)
    can be considered equal to the micrograms per
    hour dose of TTS-fentanyl. In one study, most
    people had satisfactory pain profiles with frequency
    of administration of every 3 days. Only
    in 24% of subjects in the study required different
    frequency of administration varying from
    48 to 60 hours.
How well did you know this?
1
Not at all
2
3
4
5
Perfectly
11
Q
  1. Which of the following is true with respect to
    central pain syndromes?
    (A) The most common cause of central pain
    state are lesions located in the brainstem
    (B) The Wallenberg syndrome (lateral
    medullar syndrome) is characterized by
    contralateral facial sensory loss and
    Horner syndrome
    (C) The most common lesions that produce
    thalamic pain syndrome are infarctions
    (D) Spinal cord lesions rarely cause sensory
    deficits
    (E) Central pain syndromes of spinal origin
    usually respond to epidural steroids
A
  1. (C)
    A. The most common cause of central pain
    states are spinal cord lesions.
    B. The Wallenberg syndrome is usually vascular
    in origin, and characterized by crossed
    sensory findings that include ipsilateral
    facial sensory loss, Horner syndrome, and contralateral body impairment of pain and
    temperature loss.
    C. The most common lesions that produce thalamic
    pain syndrome are infarctions, followed
    by arteriovenous malformations (AVMs),
    neoplasms, abscesses, plaque of multiple
    sclerosis, traumatic injury, and others.
    D. Spinal cord lesions are the most common
    cause of central pain syndromes and present
    with areas of sensory loss resulting
    from disruption of the spinothalamic tract.
    E. The treatment of central pain of spinal origin
    is complex with poor response to most
    forms of therapy.
How well did you know this?
1
Not at all
2
3
4
5
Perfectly
12
Q
343. Peripheral neuropathy is a common pain syndrome
characterized by which of the following?
(A) Asymmetric paresthesias and proximal
motor impairment
(B) Proximal more than distal sensory
impairment
(C) Most peripheral neuropathies may be
classified as demyelinating, axonal, or
mixed
(D) Peripheral mononeuropathy is the most
common peripheral nerve disease in
patients with long-standing diabetes
mellitus
(E) Nerve conduction studies only measure
conduction through small unmyelinated
fibers, so impairment of the fast
conducting fibers may go undetected
A
  1. (C) Sensory symmetric impairment is commonly
    seen distally with progression to more
    proximal areas of the limbs as the disease progresses.
    Peripheral polyneopathy is the most
    common initial manifestation of diabetes mellitus.
    The nerve conduction studies measure
    only the fastest conducting fibers, leaving
    injury of small-diameter fibers, which transmit
    pain sensations, undiagnosed.
How well did you know this?
1
Not at all
2
3
4
5
Perfectly
13
Q
  1. Events seen in the development of neuropathic
    pain are
    (A) following nerve injury, there is a
    decreased activity of the sodium channels
    which allows for abnormal conduction
    through pain facilitating fibers
    (B) wide dynamic range neurons in the
    dorsal horn respond with increased
    frequency as the intensity of the
    repeated afferent stimulus increases
    (C) an increase in potassium channels would
    facilitate an amplified afferent activity
    (D) C-polymodal nociceptors are activated
    by low-threshold mechanical, thermal,
    and chemical stimuli
    (E) γ-aminobutyric acid (GABA) and
    glycine are released in the dorsal horn
    and augment the response of second
    order neurons
A
  1. (B) Following nerve injury there is an increase
    in the expression of sodium channels in the
    neuroma and in the DRG. Consistent with the
    role of sodium channels in the development of
    neuropathic pain is blockage of their activity by
    low plasma concentrations of lidocaine. A reduction
    in potassium channel activity leads to
    increased afferent activity. The largest population
    of afferent axons is C-polymodal nociceptors
    that are activated by high-threshold
    mechanical, thermal, and chemical stimuli.
How well did you know this?
1
Not at all
2
3
4
5
Perfectly
14
Q
  1. Examples of neuropathic pain conditions include
    all, EXCEPT
    (A) complex regional pain syndrome (CRPS)
    (B) diabetic peripheral neuropathy
    (C) postherpetic neuralgia (PHN)
    (D) Raynaud phenomenon
    (E) phantom limb pain
A
  1. (D) Raynaud phenomenon is not a neuropathic
    pain condition, but rather a vascular condition
    (although, potentially sympathetically mediated
    and/or sustained).
How well did you know this?
1
Not at all
2
3
4
5
Perfectly
15
Q
  1. Which of the following conditions is more
    likely to be associated with neuropathic pain?
    (A) Traumatic nerve injury
    (B) Stroke
    (C) Syringomyelia
    (D) Multiple sclerosis
    (E) Large myelinated fiber neuropathy
A
  1. (C) Although not completely known some conditions
    predispose to the development of neuropathic
    pain. The relative frequency is 5% for
    patients with traumatic nerve injury, 8% for
    patients after stroke, 28% for patients with multiple
    sclerosis, and 75% for patients with
    syringomyelia. Neuropathies with predominant involvement of large myelinated fibers are usually
    not painful.
How well did you know this?
1
Not at all
2
3
4
5
Perfectly
16
Q
  1. A patient with CRPS responds well to sympathetic
    ganglion block. The results of this block
    can lead you to say which of the following
    about this particular pain condition?
    (A) It is vascularly mediated
    (B) It is sympathetically mediated
    (C) It is sympathetically maintained
    (D) It is less severe than previously thought
    (E) It will not respond well to spinal cord
    stimulation
A
  1. (C) We do not know if it is sympathetically
    mediated (B) from the block since this does not
    provide evidence of etiology. We do not know
    the involvement of vascularity since the block
    is affecting sympathetic outflow and precludes
    vascular evidence (which could be mediated by
    a host of other physiologic events). There is no
    clinical evidence to support a less severe case (D)
    and, the evidence suggests that it will respond
    to spinal cord stimulation (E).
How well did you know this?
1
Not at all
2
3
4
5
Perfectly
17
Q
348. Neuropathic pain can result in which of the
following condition?
(A) Central sensitization
(B) Allodynia
(C) Hyperalgesia
(D) B and C
(E) A, B, and C
A
  1. (E) Central sensitization is the reason for many
    of the symptoms including allodynia and
    hyperalgesia. Therefore, all are correct.
How well did you know this?
1
Not at all
2
3
4
5
Perfectly
18
Q
  1. Potential neurophysiologic mechanisms underlying
    the development of neuropathic pain
    include
    (A) microglial activation in the spinal cord
    (B) cytokine production in the spinal cord
    (C) decreased glutamate release in the
    spinal cord
    (D) A and C
    (E) A and B
A
  1. (E) Cytokines are inflammatory mediators
    released by a variety of cells that regulate the
    inflammatory response. Systemic or local injection
    of cytokines in animal models causes mechanical
    and thermal hyperalgesia. Cytokines may cause
    excitation of nociceptors via the release of other
    mediators, like prostaglandins. At the level of the
    CNS, cytokines may be liberated by microglial
    cells. The best studied excitatory amino acid is
    glutamate. Glutamate may bind to ionotopic or
    metabotropic glutamate receptors. Peripheral and
    central activation of those receptors induces pain
    behaviors in animals. All basic science evidence
    suggests (A) and (B), but does not suggest (C).
How well did you know this?
1
Not at all
2
3
4
5
Perfectly
19
Q
350. When the stimulus of light touch exerts pain
which of the following is exhibited?
(A) Hyperalgesia
(B) Allodynia
(C) Hypereflexemia
(D) Paresthesia
(E) Hypertouchemia
A
  1. (B) Following tissue damage, there is a decrease
    of the threshold for noxious stimuli (hyperalgesia),
    which may be associated to perception of
    pain to normally innocuous stimuli. This phenomenon
    is termed allodynia. Allodynia is most
    likely caused by plastic changes at the level of the
    primary sensory fibers and spinal cord neurons.
How well did you know this?
1
Not at all
2
3
4
5
Perfectly
20
Q
351. Phantom pain refers to
(A) any sensation of the missing limb,
except pain
(B) painful sensations referred to the
missing limb
(C) spontaneous movement of the stump
ranging from small jerks to visible
contractions (jumpy stump)
(D) pain referred to the amputation stump
(E) B and D
A
  1. (B) Phantom sensation: any sensation of the
    missing limb, except pain (A).
    Stump contractions: spontaneous movement
    of the stump ranging from small jerks to
    visible contractions (jumpy stump) (C).
    Stump pain: pain referred to the amputation
    stump (D).
How well did you know this?
1
Not at all
2
3
4
5
Perfectly
21
Q
  1. A 74-year-old male has a left lower extremity
    amputation after a long bout with uncontrolled
    diabetes mellitus (DM). What are the chances
    that this patient will develop phantom pain?
    (A) 33%
    (B) 49%
    (C) 55%
    (D) 90%
    (E) 75%
A
  1. (E) While ranges between 2% and 88% are
    quoted in the literature, most current studies state that between 60% and 80% of patients
    will develop phantom pain after amputation.
How well did you know this?
1
Not at all
2
3
4
5
Perfectly
22
Q
  1. A vascular surgeon consults the pain team on
    a patient who is scheduled to undergo an
    amputation secondary to peripheral vascular
    disease. The patient has read about phantom
    pain on the Internet and would like to know
    when it would likely start. You tell the vascular
    surgeon that
    (A) the onset of phantom pain is usually
    within the first week after amputation
    (B) most studies have shown that phantom
    pain will start between 2 and 4 weeks
    after an amputation for peripheral
    vascular disease
    (C) the likelihood of her developing
    phantom pain in the first 6 months after
    amputation is low, but increases drastically
    between 6 and 9 months
    (D) the onset will likely be delayed for years
    (E) none of the above
A
  1. (A) Prospective studies in patients undergoing
    amputation mainly because of peripheral vascular
    disease have shown that the onset of
    phantom pain is usually within the first week
    after amputation.
    However, in a retrospective study of individuals
    who were congenital amputees or
    underwent amputation before the age of 6 years,
    Melzack and coworkers found that the mean
    time for onset of phantom limb pain was 9 years
    in the group of congenital amputees and 2.3 years
    in the group of individuals with early amputations.
    [Jensen TS, Krebs B, Nielsen J, et al.
    Phantom limb, phantom pain, and stump pain
    in amputees during the first 6 months following
    limp amputation.
How well did you know this?
1
Not at all
2
3
4
5
Perfectly
23
Q
  1. The patient mentioned in the previous question
    develops early and severe phantom pain:
    (A) The patient is more likely to suffer from
    long-standing pain
    (B) The patient is less likely to suffer from
    long-standing pain
    (C) The patient is more likely to suffer incapacitating
    pain for 1 year that will
    subside rather abruptly
    (D) It is likely that the patient will develop
    neuropathic pain in the extremity
    contralateral to the amputation
    (E) The pain will likely be refractory to
    treatment with anticonvulsants
A
  1. (A) Patients who develop early and severe
    phantom pain are more likely to suffer from
    chronic pain, whereas individuals who are
    pain-free at the beginning are less likely to
    develop significant pain. However, prospective
    studies with a maximum follow-up period of
    2 years suggest that phantom pain may diminish
    with time.
How well did you know this?
1
Not at all
2
3
4
5
Perfectly
24
Q
355. The number of amputees who have severe
phantom limb pain is
(A) 20% to 30%
(B) 60% to 80%
(C) 5% to 10%
(D) 1% to 2%
(E) 45% to 55%
A
  1. (C) While phantom limb pain is seen in 60% to 80% of amputees, only 5% to 10% have severe pain.
How well did you know this?
1
Not at all
2
3
4
5
Perfectly
25
356. Preamputation pain (A) is more likely to lead to phantom pain if the amputation is traumatic (B) may sensitize the nervous system, explaining why some individuals may be more susceptible to development of chronic phantom pain (C) is more likely to lead to phantom pain if the amputation is secondary (D) is similar in character and localization to the subsequent phantom pain in 80% of patients (E) is less likely to lead to phantom pain if the amputation is in the upper extremities
356. (B) Some retrospective studies, but not all have pointed to preamputation pain as a risk factor for phantom pain. It has been hypothesized that preoperative pain may sensitize the nervous system, explaining why some individuals may be more susceptible to development of chronic pain. A. It has been noted that patients with traumatic amputations, who had no pain prior to the amputation, develop pain to the same extent as patients with preoperative pain who endure amputations after significant medical pathology. C. There is no correlation between the development of phantom pain and whether the amputation was primary or secondary. Primary amputation is when the limb is lost at the time of the injury. Secondary amputation is when the limb is surgically removed in a hospital. D. Phantom pain may mimic preamputation pain in both character and localization. Preamputation pain may persist in some patients, but it is not the case in the majority of patients. E. Site of amputation has not been found to have a role in determining whether preamputation pain leads to phantom pain.
26
357. A 25-year-old left lower extremity amputee returns from Iraq. He experiences phantom pain, but is attempting to move forward in life. To ease his transition back into society which of the following is the next best step? (A) He should take as long as possible to grieve before he finds new employment (B) He should initially use a cosmetic prosthesis before embarking on the task of learning to use a functional one (C) He should absolutely refuse to ever have spinal anesthesia as it may worsen phantom pain (D) He should learn coping strategies as phantom pain is a psychological disturbance (E) None of the above
357. (E) A. Amputees who experienced a long delay between the amputation and return to work, had difficulty in finding suitable jobs, and had fewer opportunities for promotion. B. The use of a functionally active prosthesis as opposed to a cosmetic prosthesis may reduce phantom pain. C. Spinal anesthesia in amputees may precipitate transient, difficult to treat phantom pain. Given the low incidence of recurrent phantom limb pain with spinal anesthesia, its transient nature, and the fact that it can be treated if it occurs, it has been concluded that spinal anesthesia is not contraindicated in patients with previous lower limb amputation. D. While there is no evidence that phantom pain represents a psychological disturbance, it may be triggered and precipitated by psychosocial factors. It has been shown that coping strategies are important for the experience of phantom pain Research has indicated that the way individuals cope with pain may influence pain, and physical and psychological adjustment.
27
358. Stump pain and phantom pain are often confused. There are, however, notable differences. Which of the following is true? (A) Unlike phantom pain, stump pain occurs in the body part that actually exists, in the stump that remains (B) Stump pain typically is described as a “sharp,” “burning,” “electric-like,” or “skin-sensitive” pain (C) Stump pain is usually caused by a neuroma (D) Surgical revision of the stump or removal of the neuroma is sometimes considered when treating stump pain (E) All of the above
358. (E) Stump pain is located at the end of an amputated limb’s stump. Unlike phantom pain, it occurs in the body part that actually exists, in the stump that remains. It typically is described as a “sharp,” “burning,” “electric-like,” or “skinsensitive” pain. Some patients have spontaneous movements of the stump, ranging from slight, hardly visible jerks to severe contractions. Stump pain results from a damaged nerve in the stump region. Nerves damaged in the amputation surgery try to heal and may form abnormally sensitive regions, called neuromas. A neuroma can cause pain and skin sensitivity. Percussion of neuromas may increase nerve fiber discharge and augmentation of stump and phantom pain. No one treatment has been shown to be effective for stump pain. Because it is a pain caused by an injured peripheral nerve, drugs used for nerve pain may be helpful. If the stump pain affects a limb, revision of the prosthesis is sometimes beneficial. Other approaches also are tried in selected cases, including: nerve blocks, transcutaneous electrical nerve stimulation, surgical revision of the stump, or removal of the neuroma (this procedure may fail because the neuroma can grow back; some patients actually get worse after surgery), and cognitive therapies. Stump pain is common in the early postamputation period. Stump pain can also persist beyond the stage of postsurgical healing. Stump pain and phantom pain are strongly correlated. Phantom pain subsides with resolution of stump pain and that it is more prevalent in patients with phantom pain than in those without it. Careful sensory examination of amputation stumps may reveal areas of sensory abnormalities such as hypoesthesia, hyperalgesia, or allodynia. However, a correlation between phantom pain and the extent and degree of sensory abnormality has not been established.
28
359. A neuroma is an inflammation of a nerve that is seen universally after a nerve has been cut (ie, during an amputation). They show spontaneous and abnormal evoked activity following mechanical or chemical stimulation from the periphery. This results from (A) an increased and novel expression of sodium channels (B) hyperexcitability changes and reorganization of the thalamus (C) an increase in potassium efflux (D) increased activity in afferent C fibers (E) A and D
359. (E) The ectopic and increased spontaneous and evoked activity from the periphery is assumed to be the result of an increased and also novel expression of sodium channels. Local anesthesia of the stump may reduce or abolish phantom pain temporarily. Decreasing peripheral output by locally anesthetizing stump neuromas with lidocaine reduced tap-evoked stump pain. On the other hand, there was a clear increase in pain when the potassium channel blocker, gallamine was injected in the perineuromal space. Both findings support the premise that abnormal input from peripheral nociceptors plays a role in pain generation.
29
360. Some amputees show an abnormal sensitivity to pressure and to repetitive stimulation of the stump, which can provoke attacks of phantom pain. Which of the following is the case in humans? (A) It can be reduced by giving the NMDA antagonist, ketamine (B) It can only be reduced by terminating the stimulation (C) It can be attributed to the general excitability of spinal cord neurons, where only C fibers gain access to secondary pain-signaling neurons (D) Sensitization of the dorsal horn may be mediated by glycine and serotonin (E) All of the above
360. (A) The pharmacology of spinal sensitization entails an increased activity in NMDA receptor–operated systems, and many aspects of the central sensitization can be reduced by NMDA receptor antagonists In amputees, the evoked pain from repetitive stimulation can be reduced by the NMDA antagonist ketamine B. Terminating the stimulation is not the only way to reduce the pain. C. After a nerve is injured, there is an increase in the general excitability of spinal cord neurons, where C fibers and A-δ afferents gain access to secondary pain-signaling neurons. D. Sensitization of dorsal horn neurons is mediated by release of glutamate and neurokinin. This sensitization may present in several ways including: lowered threshold, increased persistent neuronal discharges with prolonged pain after stimulation, and expansion of peripheral receptive fields. The central sensitization may also be a result of a different type of anatomical reorganization. Substance P is normally expressed in small afferent fibers, but following nerve injury, it may be expressed in large A-β fibers. This phenotypic switch of large A-β fibers into nociceptive-like nerve fibers may be one of the reasons why nonnoxious stimuli can be perceived as painful
30
``` 361. Of the following, which does not play a role in the mechanism for generating phantom pain? (A) Peripheral sensitization (B) Central sensitization (C) Cortical reorganization (D) Increased thalamus response to stimulation (E) Sympathetic inhibition ```
361. (E) E. The sympathetic nervous system may play a role in generating and, in uparticular, in maintaining, phantom pain. After limb amputation and deafferentation in adult monkeys, there is reorganization of the primary somatosensory cortex, and while these changes may be unique to the cortex, they may also be, at least in part, the result of changes at the level of the thalamus and perhaps even brain stem or spinal cord. After dorsal rhizotomy, the threshold to evoke activity in the thalamus and cortex decreased, and the mouth and chin invade cortices corresponding to the representation of arm and fingers that have lost their normal afferent input. In humans similar reorganization has been observed. In the thalamus, neurons that normally do not respond to stimulation in amputees begin to respond and show enlarged somatotropic maps. A cascade of events seems to be involved in generating phantom pain and it starts in the periphery, spinal cord, brain stem, thalamus, and finally ends in the cerebral cortex.
31
362. Pharmacologically treating phantom pain is not easy. Which of the following medications has not proven to be effective in well-controlled trials? (A) Tramadol (B) Gabapentin (C) Memantine (D) Amitriptyline (E) A and C
362. (C) A. and D. Tramdol and amitriptyline have been found to be efficacious in treating phantom and stump pain in treatment naive patients. B. Gabapentin has been noted to be better than placebo in reducing phantom pain. Failure to pharmacologically provide pain relief should not be accepted until opioids have been tried. Intravenous (IV) and oral morphine have been shown to decrease phantom pain. Case reports have indicated that methadone may also be helpful. Other trials have not reported the same the success with an oral NMDA antagonist, memantine. Suggestions for the treatment of postamputation pain (no evidence) (Note: it is important to differentiate between early postoperative pain and chronic pain [pain persisting more than 4 weeks], and stump and phantom pain): Early postoperative pain Stump pain Conventional analgesics • Acetaminophen • NSAIDs • Opioids +/− combined with epidural pain treatment Stump and phantom pain If neuropathic pain clearly exists (paroxysms or abnormal stump sensitivity)—trial with TCAs or anticonvulsants. Chronic pain Stump pain • Local stump surgery: if obvious stump pathology is present, revisions should be considered; surgery should be avoided in cases of sympathetically maintained pain. • Local medical treatment: topical lidocaine or capsaicin can be tried in those who have stump pain but no obvious stump pathology. Stump and phantom pain (medical treatment, in order of preference) • Gabapentin 1200 to 2400 mg/d, slow titration. Max dose of 3600 mg/d. • TCAs (imipramine, amitriptyline, nortriptyline) 100 to 125 mg/d, slow titration. Check electrocardiogram (ECG) before starting. Monitor plasma levels with dose greater than 100 mg/d. If sedation is wanted, amitriptyline should be used. • If the pain is mostly paroxysmal, lancinating, or radiating: • Oxcarbazepine 600 to 900 mg/d. Start at 300 mg and increase by 300 mg daily. • Carbamazepine 450 mg/d. Start dose 150 mg, daily increments of 150 mg. Monitor plasma levels after 10 days on maximum dose. • Lamotrigine 100 to 200 mg/d. Start dose 25 mg/d, slow titration with increments of 25 mg/14 days (to avoid rash). • Opioids (long-acting) or tramadol. • If none of the above has an effect, refer the patient to the pain clinic. • In pain center: can perform IV lidocaine trial or ketamine trial. If the lidocaine test is positive—reconsider anticonvulsants. If the ketamine test is positive: consider memantine or amantadine. Physical therapy encompassing massage, manipulation, and passive range of motion may prevent trophic changes and vascular congestion in the stump. Transcutaneous electrical nerve stimulation, acupuncture, ultrasound, and hypnosis, may have a beneficial effect on stump and phantom pain.
32
``` 363. A65-year-old Vietnam War veteran with a left below the knee amputation and phantom pain has surgery on an amputation neuroma. He should expect (A) excellent resolution of his phantom pain (B) short-term pain relief (C) a likely infection and subsequent complicated hospital course (D) decreased pain only if he receives a 40-minute infusion of diphenhydramine within 24 hours of the surgery (E) none of the above ```
363. (B) Surgery on amputation neuromas and more extensive amputation were accepted treatment modalities for stump and phantom pain in the past. Today, stump revision is probably done only in cases of obvious stump pathology, and in properly healed stumps there is almost never an indication for proximal extension of the amputation because of pain. Surgery should be avoided in cases of sympathetically maintained pain. Surgery may produce short-term pain relief but pain often reappears. The results of other invasive procedures such as dorsal root entry zone lesions sympathectomy and cordotomy have generally been nontherapeutic, and most of them have been abandoned.
33
364. Apatient has tingling sensations in a phantom limb that are uncomfortable and annoying but do not interfere with activities or sleep. According to the Sunderland classification of patients with phantom pain, what group is this patient in? (A) Group I (B) Group II (C) Group III (D) Group IV (E) None of the above
364. (B) Classification of patients with phantom pain: Group I: Mild intermittent paresthesias that do not interfere with normal activity, work, or sleep. Group II: Paresthesias that are uncomfortable and annoying but do not interfere with activities or sleep. Group III: Pain that is of sufficient intensity, frequency, or duration to be distressful; however, some patients in this group have pain that is bearable, that intermittently interferes with their lifestyle, and that may respond to conservative treatment. Group IV: Nearly constant severe pain that interferes with normal activity and sleep.
34
``` 365. The gate-control theory of pain has been used to explain phantom limb pain. It states that (A) following significant destruction of sensory axons by amputation, wide dynamic range neurons are freed by inhibitory control (B) self-sustaining neuronal activity may occur in spinal cord neurons (C) if spontaneous spinal cord neuronal activity increases by any amount, pain may occur in the phantom limb (D) A and B (E) A, B, and C ```
365. (D) A. and B. The gate control theory of pain, put forward by Ronald Melzack and Patrick David Wall in 1962, and again in 1965, is the idea that the perception of physical pain is not a direct result of activation of nociceptors, but instead is modulated by interaction between different neurons, both paintransmitting and non–pain-transmitting. The theory asserts that activation of nerves that do not transmit pain signals can interfere with signals from pain fibers and inhibit an individual’s perception of pain. It has been used to explain phantom limb pain. Following marked destruction of sensory axons by amputation, wide dynamic range neurons are freed by inhibitory control. Self-sustaining neuronal activity may then occur in spinal cord neurons. C. If the spontaneous spinal cord neuronal activity exceeds a critical level, pain may occur in the phantom limb. This loss of inhibitory control may lead to spontaneous discharges at any level in the CNS and may explain the lack of analgesia in paraplegics with phantom body pain after complete cordectomy Pain increases after blocking conduction are in line with the theory, as continued loss of peripheral sensory input would lead to further disinhibition. Sodium thiopental perpetuates CNS inhibition and has been reported to end phantom limb pain during spinal anesthesia. Melzack R, Wall PD. Mechanisms: a new theory. Agate control system modulates sensory input from the skin before it evokes pain perception and response. Science. 1965;150(3699).
35
366. All of the following are true about primary dysmenorrhea, EXCEPT (A) pain is transmitted via the thoracolumbar spinal segments and pelvic afferents (B) the etiology of pain includes myometrial contractions leading to intense intrauterine pressure and uterine hypoxia (C) prostaglandins and leukotriene production that sensitizes afferent pelvic nerves is part of its pathogenesis (D) endometriosis and adenomyosis are its most common causes (E) altered central receptivity of the afferent input from the pelvis is thought to be relevant in its development
366. (D) Primary dysmenorrhea is defined as menstrual pain without pelvic pathology. Endometriosis and adenomyosis are the most common causes of secondary dysmenorrhea.
36
``` 367. All of the following are true about chronic endometriosis, EXCEPT (A) ovaries, cul-de-sac, uterine tubes, surface of the bowel are among the most common sites of pathologic implantation of the functioning endometrial tissue (B) retrograde menstruation, lymphatic spread, and hematogenous spread of the endometrial tissue are all thought to play a role in endometriosis etiology (C) pain occurs only with menses (D) definitive diagnosis can be made by visualization of the characteristic lesions without a mandatory histologic confirmation (E) leuprolide acetate (Lupron) may be an effective treatment of the symptoms of chronic endometriosis ```
367. (C) The pain of endometriosis can occur with menses or sexual intercourse or can always be present. It can also mimic any known pelvic pathology. Answers A, B, D, and E are all correct.
37
368. All of the following are correct, EXCEPT (A) pudendal nerve takes origin from S2, S3, and S4 roots bilaterally (B) bilateral denervation of the inferior hypogastric nerves is as effective as a lumbar epidural block with respect to sensory input from the uterus and cervix (C) many patients with hymenal neuropathy are so emotional and complain so violently that the pelvic examination is not possible (D) patients with sympathetic pelvis syndrome have a deep pain in the pelvis not associated with physically detectable abdominal wall or muscle tenderness (E) ilioinguinal and iliohypogastric neuropathy is rarely associated with the surgeries in the lower abdominal wall area
368. (E)
38
369. All of the common reasons for the inadequate management of acute pain in a hospital setting are true, EXCEPT (A) the common idea that pain is merely a symptom and not harmful in itself (B) the fact that opioids have no potential for addiction when administered strictly for acute pain (C) lack of understanding of the pharmacokinetics of various agents (D) lack of appreciation of variability in analgesic response to opioids (E) prescription of inappropriately low doses of opioids and thinking that opioids must not be given more often than every 4 hours
369. (B) Opioids have the potential for addiction even when administered for acute pain. However, it is the exaggerated common fear of the potential for addiction to opioids that often interferes with adequate pain management. The rest of the answers are correct.
39
370. The following are true about pathologic (nonphysiological) pain, EXCEPT (A) it occurs in the context of central sensitization (B) it occurs in the context of peripheral sensitization (C) it outlasts the stimulus (D) it spreads to nondamaged areas (E) it is elicited by A-δ and C fibers, but not A-β fibers, which transmit touch sensation
370. (E) It is recognized that long-term changes occur within the peripheral and central nervous system following noxious input. This neuroplasticity alters the body’s response to usual peripheral sensory input. In pathologic pain conditions, stimulation of A-β fibers, normally eliciting response to touch, may elicit pain.
40
371. Perioperative administration of NSAIDs (A) does not reduce the demand for opioids during and after the surgery (B) is contraindicated because of increased possibility of bleeding (C) has synergistic effect with opioids (D) has its analgesic effect only through peripheral mechanisms (E) is not associated with the concerns for postoperative bleeding
371. (C) Even though there have been some concerns regarding the risks of perioperative NSAIDs, including intra- and postoperative bleeding, they continue to have a useful role. Combination of NSAIDs and opioids has a synergistic analgesic effect, as they act at the different sites of pain pathways. More new evidence is emerging that NSAIDs exert their analgesic effects also through the central mechanisms.
41
``` 372. All of the following are true about the NMDA receptors, EXCEPT (A) they are involved in development of “windup” facilitation (B) NMDA agonists reduce development of tolerance to opioids (C) NMDA receptors are involved in development of central sensitization (D) NMDA receptors are involved in changes of peripheral receptive fields (E) NMDA receptors are involved in induction of oncogenes and long-term potentiation ```
372. (B) It has been demonstrated that the administration of an NMDA antagonist reduces the development of tolerance to morphine. The rest of the answers are correct.
42
373. As compared with somatic pain, all of the following are true about visceral pain, EXCEPT (A) it may follow the distribution of a somatic nerve (B) it is dull and vague (C) it is often periodic and builds to peaks (D) it is often associated with nausea and vomiting (E) it is poorly localized
373. (A) The following are the usual features of the somatic pain: well localized, sharp and definite, often constant (sometimes periodic); it is rarely associated with nausea usually when it is deep somatic pain with bone involvement; it may be following the distribution of a somatic nerve. In contrast, the visceral pain: is poorly localized, diffuse, dull, and vague; it is often periodic and builds to peaks (sometimes constant); it is often associated with nausea and vomiting.
43
``` 374. The following statements are true regarding preemptive analgesia, EXCEPT (A) preemptive analgesia is helpful in reducing postoperative pain in part by reducing the phenomenon of central sensitization (B) early postoperative pain is not a significant predictor of long-term pain (C) local anesthetics, opioids, and NSAIDs can be used for preemptive analgesia (D) preemptive analgesia may have the potential to prevent the development of chronic pain states (E) preemptive analgesia is thought to reduce neuroplastic changes in the spinal cord ```
374. (B) It has been demonstrated that early postoperative pain is a significant predictor of longterm pain. The rest of the answers are correct.
44
375. The following statements are true regarding multimodal analgesia, EXCEPT (A) it may include NSAIDs, acetaminophen, local anesthetics, and opioids in the same patient (B) it is beneficial because of the synergistic action of the individual medications with different sites of action along the pain pathways (C) it is not very valuable owing to an increase in the incidence of side effects (D) it facilitates early mobilization of the postsurgical patient (E) it expedites return to normal parenteral nutrition
375. (C) Multimodal analgesia makes it possible to significantly reduce the total consumption of opioids intra- and postoperatively. Therefore, opioid side-effects are minimized, including inevitable opioid-induced GI stasis that delays the resumption of normal enteral nutrition after surgery.
45
``` 376. All of the following statements about PHN are correct, EXCEPT (A) midthoracic dermatomes is one of the most common sites for PHN (B) men are affected more often than women in a ratio of 3:2 (C) ophthalmic division of the trigeminal nerve is one of the most common sites for PHN (D) PHN may occur in any dermatome (E) PHN has an incidence of 9% to 14.3% ```
376. (B) PHN affects women more often than men, in a ratio of approximately 3:2. The rest of the answers are correct.
46
``` 377. PHN is defined as (A) any pain associated with the herpes zoster (B) pain caused by herpes zoster for more than 1 month (C) persistent pain with a significant neuropathic component in a dermatomal distribution (D) pain caused by herpes zoster for more than 3 months (E) neuropathic pain in midthoracic dermatomes caused by herpes simplex virus ```
377. (B) PHN is defined as pain caused by herpes zoster for more than 1 month.
47
``` 378. Which of the following is true about the management of PHN? (A) Approximately 40% of patients with PHN have either incomplete or no relief from treatment (B) Prevention of herpes zoster is not nearly as important as a multimodal treatment of PHN (C) Current multimodal treatment of PHN is nearly 100% effective, independent of the duration of the symptoms (D) Current multimodal treatment of PHN is nearly 100% effective as long as it is started within the first month of the symptoms of PHN (E) Current multimodal treatment of PHN is nearly 100% effective as long as it is started immediately after the first symptoms of herpes zoster ```
378. (A) As many as 40% of patients with PHN have either incomplete or no relief from treatment. Because of this, the future may lie with prevention through vaccination and early aggressive treatment of herpes zoster with antivirals and analgesics to reduce the extent of the nerve damage and sensitization that may correlate with PHN.
48
379. The following are true about the use of antidepressants in treatment of PHN, EXCEPT (A) amitriptyline has been shown to be effective in treatment of PHN, but has significant limitations in the long term because of its side effects (B) selective serotonin reuptake inhibitors (SSRIs) have been found to be equally or more effective in treatment of PHN than the older generation of tricyclic antidepressants (TCAs) or selective norepinephrine reuptake inhibitors (SNRIs) (C) SNRIs have been shown to be more effective than placebo in treatment of PHN (D) antidepressant therapy in PHN is built on sound, scientific basis (E) one of the significant side effects of TCAs is their anticholinergic properties
379. (B) Experience with serotonergic antidepressants, such as clomipramine, trazodone, nefazodone, fluoxetine, and zimelidine, in PHN has been disappointing. The evidence supporting the use of noradrenergic agents is more compelling. The rest of the answers are correct.
49
380. Which of the following is true about use of opioids in the treatment of PHN? (A) The use of opioids is not justified for nonmalignant pain (B) Opioids tend to be less effective for the treatment of neuropathic pain than nonneuropathic pain (C) Opioids were not found to be useful in the treatment of PHN (D) The use of opioids should be avoided in combination with antidepressants because of the risk of excessive central nervous system (CNS) suppression (E) The use of opioids in PHN should be avoided owing to the increased potential of addiction
380. (B) There has been evidence that opioids do not relieve neuropathic pain as well as nonneuropathic pain. However, there is also evidence that opioids have been successfully used for the treatment of PHN.
50
``` 381. Which of the following is the most common cause of autonomic neuropathy in the developed world? (A) Leprosy (B) Diabetes mellitus (DM) (C) Human immunodeficiency virus (HIV) infection (D) Heavy metal poisoning (E) Idiopathic etiology ```
381. (B) DM is the most common cause of autonomic neuropathy, and peripheral neuropathy in general, in the United States, as well as in the rest of the developed world. Leprosy is the most common cause of peripheral neuropathy in the world.
51
382. Diabetic amyotrophy (A) has a poor prognosis (B) has better prognosis when it involves upper extremities (C) usually resolves within 1 to 2 years spontaneously (D) has better prognosis when the symptoms do not involve pain (E) it is directly related to hyperglycemia
382. (C) Diabetic amyotrophy starts with pain and involves the lower extremities. It has a good prognosis and usually resolves spontaneously in 12 to 24 months. It is not directly related to hyperglycemia.
52
``` 383. The following are true about the distal sensorimotor polyneuropathy, EXCEPT (A) it is the most common neuropathic manifestation of both type 1 and type 2 diabetes (B) it starts distally and spreads proximally (C) initial symptoms may involve numbness and tingling in the toes or feet (D) it is a length-dependent neuropathy (E) it is usually asymmetrical ```
383. (E) Distal sensorimotor polyneuropathy is a symmetrical length-dependent process with dying-back or dropout of the longest nerve fibers—myelinated and unmyelinated. All other answers are correct.
53
384. The prevalence of diabetic neuropathy in DM patients is (A) less than 1% at diagnosis of DM, rising to 10% in patients diagnosed for longer than 5 years (B) about 10% at diagnosis of DM, rising to more than 50% in patients diagnosed for longer than 5 years (C) about 50% at diagnosis of DM, rising to almost 100% in patients diagnosed for longer than 5 years (D) about 50% at diagnosis of DM, and does not change significantly with time (E) no such studies have been done so far
384. (B) It is generally agreed that the prevalence of neuropathy is about 10% at diagnosis of DM, rising to 50% or more in patients diagnosed for longer than 5 years
54
385. Patients with diabetic distal sensorimotor polyneuropathy initially may complain of numbness and tingling in the toes or feet, which then slowly spreads proximally over months to years. Eventually, numbness and tingling appear in the fingertips, as the symptoms of diabetic polyneuropathy progress to (A) ankle (B) knee (C) mid-thigh (D) buttock and groin (E) abdomen
385. (B)
55
386. Which of the following is the most widely accepted cause of trigeminal neuralgia? (A) Demyelinating conditions, as trigeminal neuralgia is most common in patients with multiple sclerosis (B) Direct trauma of the trigeminal ganglion at the level of the foramen ovale, before branching into its three branches (C) Arterial cross-compression of the trigeminal nerve in the posterior fossa (D) Tumors of the posterior fossae (E) Poor vascular supply to the affected trigeminal branch
386. (C) It is accepted that the most common cause of trigeminal neuralgia is arterial crosscompression of the trigeminal nerve in the posterior fossa, as suggested by Jannetta in 1982. Electron microscopy of trigeminal nerve biopsies taken from patients with trigeminal neuralgia has shown areas of axonal swelling and demyelination adjacent to the area of arterial compression. Although trigeminal neuralgia is more common in patients with multiple sclerosis, only a small portion of patients with trigeminal neuralgia suffer from multiple sclerosis and does not explain the majority of the cases.
56
387. Which of the following is true regarding medical management for the treatment of trigeminal neuralgia? (A) Anticonvulsant medications are usually considered as the second line of treatment (B) Beneficial effects of carbamazepine are better in elderly patients (C) Risk of side effects of carbamazepine increase with age (D) Carbamazepine has proven to be the most effective treatment for trigeminal neuralgia, independently of the sideeffect profile (E) Because of the unlikelihood of serious side effects with surgery, all patients should consider this option first
387. (C) Carbamazepine is likely to be beneficial in up to 70% of the patients. Incidence of side effects is often higher in elderly patients especially if the drug escalation is too fast. Allergic rash is seen in up to 10% of the patients and high concentration of the drug may be associated with fluid retention promoting cardiac problems. Carbamazepine is a potent hepatic enzyme inducer which can potentially lead to undesirable drug-to-drug interactions. Although microsurgical exploration of the posterior fossa is the highly successful, it is a major surgery with 0.5% risk of mortality and major morbidity. The effectiveness of pimozide for trigeminal neuralgia is better than carbamazepine, but the high frequency of side effects limits its clinical use.
57
388. The gasserian ganglion (A) receives exclusively proprioceptive information from the muscles of mastication (B) the mandibular branch is located medial to the ophthalmic branch (C) the two medial branches are sensory while the lateral branch is partially motor (D) the ganglion lies out of the cranium, in the Meckel cave (E) the foramen rotundum is used as landmark for the blockage of the trigeminal ganglion
388. (C) The trigeminal ganglion receives sensation from the oral mucosa, scalp, nasal areas, face, and teeth. Proprioceptive information is transmitted into the ganglion from the mastication and extraocular muscles. The peripheral branches of the ganglion are the ophthalmic, the maxillary, and the mandibular, which are organized somatotropically, with the ophthalmic branch located dorsally, the maxillary branch is intermediate, and the mandibular nerve is located ventrally. The gasserian ganglion lies within the cranium, in the middle cranial fossa. The posterior border of the ganglion includes the dura of the Meckel cave. The landmark to perform the trigeminal ganglion block is the foramen ovale and not the foramen rotundum.
58
389. Which of the following is true regarding the diagnosis of trigeminal neuralgia? (A) The diagnosis must be confirmed with magnetic resonance imaging (MRI) to detect vascular trigeminal nerve compression (B) Sensory evoked potentials is the most sensitive test to perform the diagnosis (C) The diagnosis is clinical and tests are only necessary to rule out associated conditions (D) To accurately diagnose the condition, it is necessary to correlate clinical findings with MRI and sensory evoke potential tests (E) None of the above
389. (C) The diagnosis of trigeminal neuralgia is eminently clinical and further tests are necessary only to rule out associated conditions. When the condition is found, MRI and evoked potential testing are strongly recommended to rule out secondary causes. Clinically the onset of trigeminal neuralgia is around the age of 50 years, more common in females, almost exclusively unilateral with a paroxysmal nature.
59
``` 390. Giant cell arteritis is characterized by which of the following? (A) Affects almost exclusively Asian population (B) As other forms of vasculitis, giant cell arteritis commonly involves skin, kidneys, and lungs (C) Males are more commonly affected (D) It is more common in older patients, with a peak incidence between 60 to 75 years of age (E) Visual loss is the presenting symptom in over 50% of the patients ```
390. (D) The giant cell arteritis affects almost exclusively the white population although it can occur in worldwide. Unlike other forms of vasculitis it rarely affects skin, kidneys, or lungs. Females are affected 3 times more often than males. Visual loss is now considered to affect between 6% to 10% of patients in most series.
60
391. According to the International Headache Society Diagnostic Criteria, analgesic rebound headache is (A) headache that resolves or reverts within 2 weeks after discontinuation of the suspected medication (B) headache that worsens after intake of analgesics and reduces in intensity and frequency with reduction in the analgesic dose (C) the intensity of the headache decreases in intensity proportionally to the decrease in the dose of analgesic (D) headache greater than 15 days per month that has developed or markedly worsened during medication overuse (E) headache that increases in intensity with the use of morphine, most likely because of the cerebral vasodilation mediated by histamine release
391. (D) Analgesic rebound headache resolves or reverts to its previous pattern within 2 months of discontinuing of the overused medication.
61
392. Cluster headaches are characterized by (A) lancinating unilateral headache that is commonly triggered by stress factors (B) the pain is strictly unilateral and autonomic symptoms occur ipsilateral to the pain (C) the onset is slow with progressive worsening of the pain over several hours with an attack usually lasting 3 to 4 days (D) melatonin is commonly indicated as therapy for the acute attack (E) cluster headaches are more common in elderly patients
392. (B) The first statement better describes trigeminal neuralgia. Cluster headache affects more males than females with a 5:1 ratio and can begin at any age. Attacks are severe, stabbing, screwing, unilateral pain, occasionally preceded by premonitory symptoms, with sudden onset, and rapid crescendo. Therapeutic interventions for the acute attack include oxygen, triptans, dihydroergotamine, ketorolac, chlorpromazine, or intranasal lidocaine, cocaine, or capsaicin. Melatonin has been found to be moderately effective as a preventive treatment in episodic and chronic cluster headache
62
393. Which of the following describes the pathophysiologic changes seen in migraine? (A) Inflammation of hypothalamic structures leads to low threshold stimulation of vascular and meningeal tissues (B) Central sensitization mediated by attribution to activation of β-fibers in the trigeminal system, mediates extracranial hypersensitivity (C) Large cerebral vessels, pial vessels, large sinuses, and the dura, are innervated by fibers originating from the sphenopalatine ganglion (D) Activation and threshold reduction of the trigeminocervical complex by its most caudal cells (E) In acute attacks, a marked reduction in vasoactive substances, including substance P, calcitonin gene related peptide (CGRP), and nitric oxide is commonly seen
393. (D) Sterile neurogenic inflammation is often seen after stimulation of the trigeminal ganglion, which innervates large cerebral vessels, pial vessels, large sinuses, and the dura via unmyelinated C fibers. In acute attacks of migraine, substance P, CGRP, and nitric oxide mediate the neurogenic inflammation
63
``` 394. Which of the following is correct regarding headache? (A) Migraine is the most common form of headache (B) Tension-type headache (TTH) is commonly aggravated by physical exercise (C) The presence of nausea, vomiting, photophobia, or phonophobia excludes the diagnosis of TTH (D) The most common form of migraine is associated with aura (E) Comorbid conditions associated with chronic migraine include depression, anxiety, and panic disorders ```
394. (E) TTH is the most common type of headache. Aura is present in only 20% of patients suffering from migraine. Although chronic daily headache diagnostic criteria for probable TTH requires no nausea or vomiting as one of the criteria or absence of photophobia, or phonophobia, nausea may be seen in 4.2% of patients with TTH, while phonophobia is reported in 10.6% of them.
64
395. Hundred precent oxygen inhalation is a safe and effective method for acute treatment of (A) chronic daily headache (B) TTH (C) migraine with aura (D) cluster headache (E) glossopharyngeal neuralgia
395. (D) Inhalation of 100% oxygen at 7 to 12 L/min is effective in treating the majority of cluster headache sufferers when used continuously for 15 to 20 minutes. Generally oxygen inhalation is not considered to be effective in any other form of primary neurovascular headache.
65
396. The Ramsay Hunt syndrome is caused by the infection of the varicella-zoster virus of the (A) sphenopalatine ganglion (B) gasserian ganglion (C) geniculate ganglion (D) glossopharyngeal ganglion (E) stellate ganglion
396. (C)
66
397. Which of the following characterizes the spontaneous intracranial hypotension (SIH)? (A) Is the same entity as post–dural puncture headache (PDPH) (B) Headache is consistently unilateral (C) Orthostatic headache is pathognomonic (D) Patients complain of bitemporal headache (E) To confirm the diagnosis, it is required that cerebrospinal fluid (CSF) opening pressures be below 60 mm H2O
397. (C) PDPH and SIH are two distinct clinical entities with similar presentation. The headache is always bilateral, located in the occipital and/or frontal area. Although low CSF pressure is often noted, it is not necessary to confirm the diagnosis.
67
398. A 20-year-old male presents to the clinic with complaints of moderate headaches located bilateral in the forehead, parietal, and occipital areas. The pain is dull and continuous and not associated with nausea, vomiting, photophobia, and phonophobia. The patient recalls that the symptoms started 1 year ago and have been constant since they started. No abnormalities where observed on physical examination, sinus computed tomography (CT), or brain MRI. The patient has occasionally tried over-the-counter analgesics with no relief. Which of the following is the most likely diagnosis? (A) Status migrainosus (B) Rebound headache (C) New daily persistent headache (D) Cluster headache (E) Classical migraine
398. (C) New daily persistent headache is a chronic, unremitting headache of sudden onset, daily pattern. The duration of the headache should be at least 3 months. Some important features include its moderate severity, bilateral location, and lack of nausea, vomiting (N/V), photophobia, and phonophobia (P/P). On the other hand, status migrainosus is a severe debilitating migraine, associated with N/V, P/P, and with duration longer than 72 hours but that typically do not exceed 2 weeks. The other diagnoses are not consistent with the symptoms.
68
``` 399. Which of the following is a theory that may explain the presence of aura? (A) Cortical spreading depression (B) The vascular theory (C) Hormonal fluctuation (D) Estrogen withdrawal (E) Cerebral idiopathic hypertension ```
399. (A) The previously known classic migraine (migraine with aura) is preceded by visual aura that starts 20 to 40 minutes before the migraine and is characterized by spreading scintillations reflecting a slow propagation of neuronal and glial excitation emanating from one occipital lobe. Cortical spreading depression (CSD) presents with dramatic shifts in cortical steady potential (DC), temporary increases in extracellular ions and excitatory neurotransmitters (glutamate), and transient raise, followed by sustained decrease in cortical blood flow. The vascular theory proposed that migraine with aura is caused by intracranial cerebral vasoconstriction and the headache by reactive vasodilation. Despite that, the theory can not explain the prodromal symptoms or why some antimigraine medications are not effective. Hormonal fluctuations and estrogen withdrawal may explain the higher incidence of migraine in female patients during their reproductive years, but are not related to the presence of aura. Cerebral idiopathic hypertension is a form of headache of unknown etiology
69
400. Chronic low back pain and neck pain persists 1 year or longer in what percentage of patients? (A) 5% to 10% (B) 15% to 20% (C) 20% to 25% (D) 25% to 60% (E) 60% to 75%
400. (D) The published literature commonly states that 80% to 90% of low back pain resolves in about 6 weeks, irrespective of the administration or type of treatment, with only 5% to 10% of patients developing persistent back pain. Contrary to this assumption, actual analysis of research evidence shows that chronic low back and neck pain persist 1 year or longer in 25% to 60% of adult and/or elderly patients.
70
``` 401. The prevalence of zygapophysial (facet) joint involvement in low back pain is (A) 5% to 10% (B) 10% to 15% (C) 15% to 45% (D) 50% to 60% (E) 65% to 70% ```
401. (C) Based on evaluations utilizing controlled diagnostic blocks, the prevalence of zygapophysial or facet joint involvement has been estimated to be between 15% and 45% in heterogeneous groups of patients with chronic low back pain.
71
``` 402. A 58-year-old with metastatic lung cancer suddenly complains of severe back pain. Symptoms of early spinal cord compression include all of the following, EXCEPT (A) rapid onset (B) symmetric and profound weakness (C) spasticity (D) increased deep tendon reflexes (E) urinary retention and constipation ```
402. (E) The clinical picture of metastatic epidural spinal cord compression is uniformly reported as pain, weakness, sensory loss, and autonomic dysfunction. Metastatic epidural spinal cord compression initially presents with severe back pain in 95% of cases. After weeks of progressive pain, the patient may develop weakness, sensory loss, autonomic dysfunction, and reflex abnormalities. Bladder and bowel dysfunction are rarely presenting symptoms, but may appear after sensory symptoms have occurred. The exception to this generalization develops with compression of the conus medullaris, which presents as acute urinary retention and constipation without preceding motor or sensory symptoms
72
403. Specific indications for discography include all of the following, EXCEPT (A) further evaluation of abnormal discs to assess the extent of abnormality (B) patients with persistent, severe symptoms in whom other diagnostic tests have revealed clear confirmation of a suspected disc as the source of pain (C) assessment of patients who have failed to respond to surgical procedures to determine if there is possible recurrent disc herniation (D) assessment of discs before fusion to determine if the discs within the proposed fusion segment are symptomatic (E) assessment of minimally invasive surgical candidates to confirm a contained disc herniation or to investigate contrast distribution pattern before intradiscal procedures
403. (B) Patients with severe, persistent symptoms (discogenic in origin) that have been confirmed by other diagnostic evaluations do not need to undergo further evaluation by discography. Specific uses for discography include, but are not limited to: further evaluation of demonstrably abnormal discs to help assess the extent of abnormality or correlation of the abnormality with clinical symptoms (in case of recurrent pain from a previously operated disc and a lateral disc herniation); patients with persistent, severe symptoms in whom other diagnostic tests have failed to reveal clear confirmation of a suspected disc as the source of pain; assessment of patients who have failed to respond to surgical procedures to determine if there is painful pseudoarthrosis or a symptomatic disc in a posteriorly fused segment, or to evaluate possible recurrent disc herniation; assessment of discs before fusion to determine if the discs within the proposed fusion segment are symptomatic and to determine if discs adjacent to this segment are normal; and assessment of minimally invasive surgical candidates to confirm a contained disc herniation or to investigate contrast distribution pattern before intradiscal procedures.
73
404. The following signs and symptoms are consistently found with cervical radiculopathy, EXCEPT (A) gait disturbances (B) normal muscle tone (C) negative Babinski test (D) weak tendon reflexes (E) positive axial compression test (Spurling maneuver)
404. (A) Gait disturbances are a feature of cervical myelopathy, not radiculopathy. Other signs and symptoms of cervical radiculopathy include upper extremity sensory disturbances and muscle weakness.
74
405. All of the following are reasons associated with smoking as a risk factor for low back pain, EXCEPT (A) mineral content of the lumbar vertebrae is decreased (B) fibrinolytic disc activity is altered (C) blood flow and nutrition to the disc are diminished (D) disc pH is higher (E) increased degenerative changes of the lumbar spine
405. (D) Experimental studies have given support to the hypothesis that blood flow and nutrition to the disc are diminished in smokers, the pH of the disc is lowered, disc mineral content is lower, fibrinolytic activity is changed, and there are increased degenerative changes seen in the lumbar spine.
75
406. All of the following treatments have strong evidence to back their use when treating acute low back pain, EXCEPT (A) muscle relaxants effectively reduce low back pain (B) bed rest is effective for treating low back pain (C) continuing normal activity gives equivalent or faster recovery from acute low back pain (D) NSAIDs prescribed at regular intervals are an effective treatment for acute low back pain (E) different types of NSAIDs are equally effective at treating low back pain
406. (B) There is strong evidence from randomized controlled trials that bed rest is not effective for treating acute low back pain.
76
407. Age-related changes in the intervertebral discs include all of the following, EXCEPT (A) the dimensions of the lumbar intervertebral discs decrease with age (B) collagen lamellae of the annulus fibrosis increases in thickness (C) distinction between the nucleus pulposus and annulus fibrosis becomes less apparent (D) the nucleus pulposus is less able to transmit weight directly (E) 80% of nucleus pulposus cells in the elderly exhibit necrosis
407. (A) Narrowing of the intervertebral discs has long been considered one of the signs of pathologic aging of the lumbar spine, but recent data has shown that notion to be untrue. Large-scale postmortem analysis have shown lumbar disc height and diameter to actually increase with age. The anterior-posterior diameter increases by about 10% in females and 2% in males. Disc height has been shown to increase by about 10% in most lumbar discs.
77
``` 408. Radiculopathy is a neurologic condition associated with all of the following characteristics, EXCEPT (A) numbness (B) weakness (C) pain (D) compression of axons (E) ischemia of axons ```
408. (C) Radiculopathy is a condition in which conduction within the axons of a spinal nerve or its roots are blocked. It can result in numbness and weakness secondary to conduction block in sensory and motor neurons respectively. Conduction blockade can be caused by compression or ischemia. It is important to make the distinction that radiculopathy does not cause pain. It may, however, be associated with pain.
78
409. Adverse effects of epidurally administered steroids include all of the following, EXCEPT (A) Cushing syndrome (B) osteoporosis (C) avascular bone necrosis (D) hypoglycemia (E) suppression of the hypothalamus-pituitary axis
409. (D) The major theoretical complications of corticosteroid administration include suppression of pituitary-adrenal axis, hypercorticism, Cushing syndrome, osteoporosis, avascular necrosis of bone, steroid myopathy, epidural lipomatosis, weight gain, fluid retention, and hyperglycemia.
79
410. Relative contraindications to epidural steroid injections include (A) preexisting neurologic disorder (ie, multiple sclerosis) (B) sepsis (C) therapeutic anticoagulation (D) localized infection at injection site (E) patient refusal
410. (A) Absolute contraindications to epidural steroid injections include sepsis, infection at injection site, therapeutic anticoagulation, and patient refusal. Relative contraindications include preexisting neurologic conditions, prophylactic low-dose heparin, thrombocytopenia, and uncooperative patients.
80
411. L4-L5 disk herniation with L5 nerve root involvement includes (A) numbness over the medial thigh and knee (B) weakness with dorsiflexion of great toe and foot (C) difficulty walking on toes (D) pain in lateral heel (E) quadriceps weakness
411. (B) L4-L5 disc herniation with L5 nerve root involvement involves: pain over the sacroiliac joint, hip, lateral thigh, and leg; numbness over the lateral leg and first three toes; weakness with dorsiflexion of great toe and foot; difficulty walking on heels; possible foot drop; and internal hamstring reflex diminished or absent. Numbness over the medial thigh and knee, and quadriceps weakness are indicative of L3-L4 disc herniation with L4 nerve root involvement. Difficulty walking on toes and lateral heel pain are common with L5-S1 disk herniation involving the S1 nerve root.
81
``` 412. In patients with chronic low back pain, the prevalence of sacroiliac joint pain is (A) 10% (B) 15% (C) 20% (D) 25% (E) 30% ```
412. (B) Fifteen percent of patients with chronic low | back pain have sacroiliac joint pain.
82
413. Spondylolysis 414. Spondylolisthesis 415. Kissing spines 416. Radiculopathy 417. Radicular pain (A) Neurologic condition in which conduction is blocked to the axons of a spinal nerve or its roots. It results in numbness and weakness (B) An acquired defect caused by fatigue fracture of the pars interarticularis (C) Pain that arises as a result of irritation of a spinal nerve or its roots (D) Displacement of a vertebrae or the vertebral column in relationship to the vertebrae below it (E) Periostitis of spinous processes or inflammation of the affected ligament
413 to 417. 413 (B); 414 (D); 415 (E); 416 (A); 417 (C) Spondylolysis is an acquired defect that results from a fatigue fracture of the pars interarticularis (the part of the lamina that intervenes between the superior and inferior articular processes on each side). Spondylolisthesis is the displacement of a vertebrae or the vertebral column in relationship to the vertebrae below. Kissing spines (also known as Baastrup disease) affects the lumbar spinous processes. Excessive lumbar lordosis or extension injuries to the lumbar spine cause adjacent spinous processes to clash and compress the intervening interspinous ligament. This results in a periostitis of the spinous process or inflammation of the affected ligament. Radiculopathy is a neurologic condition in which conduction blocks the axons of a spinal nerve or its roots that results in numbness and weakness. Radicular pain is pain that arises as a result of irritation of a spinal nerve or its roots.
83
418. Evidence regarding the value of epidural injections for the management of chronic spinal pain demonstrates the following: (A) Limited with interlaminar lumbar epidural steroid injections for short-term relief of lumbar radicular pain (B) Strong with interlaminar lumbar epidural steroid injections for long-term relief of lumbar radicular pain (C) Moderate for lumbar transforaminal epidural steroid injections for short-term relief of lumbar radicular pain (D) Strong for lumbar transforaminal epidural steroid injections for long-term relief of lumbar radicular pain (E) Strong for caudal epidural steroid injections for short-term relief of lumbar radiculopathy and post–lumbar laminectomy syndrome
418. (E) In managing lumbar radicular pain with interlaminar lumbar epidural steroid injections, the evidence is strong for short-term relief and limited for long-term relief. In managing cervical radiculopathy with cervical interlaminar epidural steroid injections, the evidence is moderate. The evidence for lumbar transforaminal epidural steroid injections in managing lumbar radicular pain is strong for short-term and moderate for long-term relief. The evidence for cervical transforaminal epidural steroid injections in managing cervical nerve root pain is moderate. The evidence is moderate in managing lumbar radicular pain in post–lumbar laminectomy syndrome. The evidence for caudal epidural steroid injections is strong for short-term relief and moderate for long-term relief, in managing chronic pain of lumbar radiculopathy and post–lumbar laminectomy syndrome.
84
``` 419. All of the following statements regarding intervertebral disc innervation are true, EXCEPT (A) nerve plexuses that innervate the intervertebral discs are derived from dorsal rami (B) in normal lumbar intervertebral discs, nerve fibers are only found in the outer third of the annulus fibrosis (C) discs painful on discography and removed with operation have nerve growth deep into the annulus and into the nucleus pulposus (D) disc fissuring is a trigger for neoinnervation of a disc (E) the anterior and posterior nerve plexuses accompany the anterior and posterior longitudinal ligaments ```
419. (A) The sources of the nerve endings in the lumbar discs are two extensive microscopic plexuses of nerves that accompany the anterior and posterior longitudinal ligaments. The nerve plexuses that innervate intervertebral discs are derived from the lumbar sympathetic trunks. The dorsal rami supply innervation to the muscles of the back and zygapophysial joints. In normal lumbar intervertebral discs, nerve fibers are only found in the outer third of the annulus fibrosis. Discs painful on discography and removed with operation have nerve growth deep into the annulus and into the nucleus pulposus. Disc fissuring is a trigger for neoinnervation and neovascularization of a disc.
85
420. Three days after a lumbar epidural steroid injection was given, a 57-year-old male complains of fever and severe back pain over the site where the injection was given. Two days later, the back pain has progressively worsened, and a severe radiating pain goes down the right leg and knee. Which of the following is the most likely complication of the epidural steroid injection? (A) Epidural abscess (B) Epidural hematoma (C) Arachnoiditis (D) Anterior spinal artery syndrome (E) Cauda equina syndrome
420. (A) Development of an epidural abscess is a very rare complication of epidural steroid injections. It needs to be recognized and treated quickly to avoid irreversible injury. Symptoms of an epidural abscess include severe back pain that is followed by radicular pain 3 days later. The initial back pain may not become evident for several days after the injection.
86
``` 421. X-ray imaging is recommended for which of the following cause of low back pain? (A) Disc bulging (B) Cauda equina syndrome (C) Spondylolisthesis (D) Lateral disc herniation (E) Spinal cord tumors ```
421. (C) Plain x-rays are recommended for possible fractures, arthropathy, spondylolisthesis, tumors, infections, stenosis, and congential deformities. CT images are recommended for bone/joint pathologies, lateral disc herniations, stenosis (ie, spinal canal, neuroforaminal, lateral recess), and for those in which an MRI is contraindicated. MRI is recommended for disc herniations, spinal stenosis, osteomyelitis, tumors (ie, spinal cord, nerve roots, nerve sheath, paraspinal soft tissue), and cauda equine syndrome.
87
``` 422. Which of the following nerve root and muscle motion combinations is correct? (A) L2—leg extension (B) L3—heel walking (C) L4—toe walking (D) L5—first toe dorsiflexion (E) S1—hip flexion ```
422. (D) The L2 nerve root is involved with hip flexion, L3 with leg extension, L4 with heel walking, L5 with first toe dorsiflexion (and heel walking), and S1 with toe walking
88
423. Which of the following is the most frequent complication of a laminotomy with discectomy? (A) Recurrent disc herniation (B) Infection (C) Dural tear (D) Neural injury (E) Failed back surgery syndrome (FBSS)
423. (C) Laminotomy with discectomy has a low infection rate, statistically. The most frequent complication is a dural tear. Neural injury may occur as a result of a dural tear and may cause long-term pain and neurologic deficit. Recurrence of the herniation occurs in approximately 5% of cases. Infection and neural injury occurs in less than 0.5% of cases.
89
``` 424. Which of the following includes conservative treatment for FBSS? (A) Discectomy (B) Chemonucleolysis (C) Rehabilitation (D) Laminectomy (E) Fusion ```
424. (C) Conservative treatment is usually the first treatment of choice for patients presenting with FBSS. It consists of medical management of contributing factors (ie, depression, obesity, smoking), rehabilitation, and behavior modification (ie, alcohol or drug dependency).
90
425. Favorable prognostic indicators for patients undergoing repeated lumbosacral surgery include all of the following, EXCEPT (A) female sex (B) satisfactory outcome from prior surgeries (C) operative findings of disk herniation (D) epidural scarring requiring lysis of adhesions (E) radicular pain
425. (D) Many prognostic indicators have been implicated in patients undergoing repeat lumbosacral spine surgery. They may or may not be significant for each patient and should be taken into context for the particular patient. Women have been found to have better outcomes than men. Patients with a history of favorable outcomes from prior surgeries tend to have better outcomes as well. A history of few previous surgeries, operative/myelographic findings of disc herniation, and a history of working immediately prior to surgery are all favorable prognostic indicators. Less favorable prognostic indicators include epidural scarring that requires lysis of adhesions and pseudoarthrosis of a prior fusion.
91
``` 426. Waddell signs were developed to help identify nonorganic causes of low back pain. They include all of the following, EXCEPT (A) tenderness (B) stimulation (C) distraction testing (D) regional disturbance (E) underreaction ```
426. (E) Waddell signs are used to help diagnose nonorganic low back pain complaints. Each of the five findings is considered positive if present. Three positive findings are considered highly suggestive of a nonorganic source of pain: 1. Tenderness: does not follow dermatomal or referral patterns and is hard to localize. 2. Stimulation testing: stimulating distant sites should not cause discomfort. 3. Distraction testing: findings when testing the same site are inconsistent when the patient’s attention is distracted. 4. Regional disturbance: motor and sensory testing yield nonanatomic findings. 5. Overreaction: inappropriate verbal remarks or facial expressions, withdrawal from touch,
92
427. A25-year-old male presents with progressively worsening neck and back pain and stiffness over 4 months that improves with light exercise and warm showers. Which of the following is the most likely diagnosis? (A) Rheumatoid arthritis (B) Ankylosing spondylitis (C) Psoriatic arthritis (D) Klippel-Feil syndrome (E) Reiter syndrome
427. (B) Ankylosing spondylitis is characterized by pain and stiffness in young males (typically ages 17-35 years) more often than females. It is worse in the morning and improves with mild exercise. The pain will typically last for at least 3 months and be diffuse in nature affecting the low back and spine. Rheumatoid arthritis is an inflammatory polyarthritis that affects middleaged women more often than men. It typically presents with morning stiffness that improves as the day progresses, and the spine is not affected until late in the disease. Psoriatic arthritis is characterized by inflammation of the skin and joints that typically presents in the fourth and fifth decades of life. Klippel-Feil syndrome is a congenital disorder that is characterized by abnormal fusion of two or more bones in the cervical spine. Reiter syndrome is a reactive arthritis that is characterized by a triad of symptoms: nongonococcal urethritis, conjunctivitis, and arthritis.
93
``` 428. Which of the following is a major criteria for cervicogenic headache? (A) Bilateral head or face pain without sideshift (B) Pain is superficial and throbbing (C) Restricted neck range of motion (D) Pain relief with digital pressure to cervical vertebrae (E) Lack of relief from anesthetic blockade ```
428. (C) The three major criteria for cervicogenic headache include (1) signs and symptoms of neck involvement (precipitation of head pain by: neck movement and/or sustained awkward head positioning, by external pressure over the upper cervical or occipital region on the symptomatic side; restriction of the range of motion in the neck; ipsilateral neck, shoulder, or arm pain of a rather vague nonradicular nature or, occasionally, arm pain of a radicular nature); (2) confirmatory evidence by diagnostic anesthetic blockades; and (3) unilaterality of the head pain without sideshift. Head pain characteristics include moderate-severe, nonthrobbing, and nonlancinating pain, usually starting in the neck, episodes of varying duration, or fluctuating, continuous pain. Other characteristics of some importance: only marginal effect or lack of effect of indomethacin, only marginal effect or lack of effect of ergotamine and sumatriptan, female sex, not infrequent occurrence of head, or indirect neck trauma by history, usually of more than only medium severity.
94
``` 429. Neurogenic claudication can be distinguished from vascular claudication by which of the following? (A) Leg tightness (B) Pain alleviated with standing (C) Pain exacerbated with lumbar flexion (D) No change in pain with exercise (E) Pain exacerbated with lying supine ```
429. (D) Neurogenic claudication pain is secondary to nerve root compression rather than lack of blood supply that is seen with vascular claudication. The pain is exacerbated by standing erect and downhill walking. Improvement comes with lying supine more than lying in the prone position, sitting, squatting, and lumbar flexion. Neurogenic claudication is not made worse with biking, uphill walking, and lumbar flexion, unlike vascular claudication. It is not alleviated with standing.
95
430. Neck pain has been suggested to have a multifactorial origin. Which of the following statements regarding neck pain is true? (A) Workplace interventions are not effective at reducing neck pain (B) Normal degenerative changes in the cervical spine are a risk factor for pain (C) Physical activity does not protect against neck pain (D) Precision work does not increase the risk of neck pain (E) Social support in the workplace does not affect neck pain
430. (A) Neck pain has been suggested to have multifactorial etiologies. Risk factors for neck pain that cannot be modified include age, sex, and genetics. There is no evidence that normal cervical spine degenerative changes are a risk factor for neck pain. Modifiable risk factors for neck pain include smoking and exposure to environmental tobacco. Participation in physical activity seems to offer a protective effect. High quantitative job demands, low social support at the workplace, inactive work position, repetitive work, and meticulous work increases the risk of neck pain. There is a lack of evidence that workplace interventions are successful in decreasing neck pain in employees.
96
431. In patients with neck pain, what is more predictive at excluding a structural lesion or neurologic compression than at diagnosing any specific etiologic condition? (A) MRI (B) Discography (C) Blood tests (D) Physical examination (E) Electrophysiology
431. (D) In patients with neck pain, the physical examination is more predictive at excluding a structural lesion or neurologic compression than at diagnosing any specific etiologic condition in patients with neck pain. Other assessment tools (ie, electrophysiology, imaging, injections, discography, functional tests, and blood tests) lack validity and utility.
97
432. All of the following characteristics are associated with a poor prognosis for neck pain, EXCEPT (A) prior neck pain (B) pain resulting from an accident (C) passive coping techniques (D) middle age (E) compensation
432. (B) Most people with neck pain do not experience a complete resolution of symptoms. Between 50% and 85% of those who experience neck pain at some initial point will report neck pain again 1 to 5 years later. These numbers appear to be similar in the general population, in workers, and after motor vehicle crashes. The prognosis for neck pain also appears to be multifactorial. Younger age was associated with a better prognosis, whereas poor health and prior neck pain episodes were associated with a poorer prognosis. Poorer prognosis was also associated with poor psychologic health, worrying, and becoming angry or frustrated in response to neck pain. Greater optimism, a coping style that involved self-assurance, and having less need to socialize, were all associated with better prognosis. Specific workplace or physical job demands were not linked with recovery from neck pain. Workers who engaged in general exercise and sporting activities were more likely to experience improvement in neck pain. Postinjury psychologic distress and passive types of coping were prognostic of poorer recovery in WAD. There is evidence that compensation and legal factors are also prognostic for poorer recovery from WAD.
98
``` 433. Which of the following is the most common complication of fluoroscopically guided interlaminar cervical epidural injections? (A) Nonpositional headache (B) Vasovagal reactions (C) Increased neck pain (D) Fever (E) Dural puncture ```
433. (C) The reported complications of fluoroscopically guided interlaminar cervical epidural injections are increased neck pain (6.7%), nonpositional headaches (4.6%), insomnia the night of the injection (1.7%), vasovagal reaction reactions (1.7%), facial flushing (1.5%), fever on the night of the procedure (0.3%), and dural puncture (0.3%). The incidence of all complications per injection is 16.8%.
99
434. A 54-year-old female complains suddenly of inability to move her legs after a transforaminal epidural steroid injection. On further examination, she is found to have intact light touch sensation, sphincter disturbance, and loss of pain and temperature sensation. What is the most likely diagnosis? (A) Cauda equina syndrome (B) Epidural hematoma (C) Epidural abscess (D) Transient paraplegia (E) Anterior spinal artery syndrome
434. (E) Anterior spinal artery syndrome classically presents in older patients with abrupt motor loss, sphincter disturbance, and nonconcordant sensory examination with preservation of sensation to light touch but loss of pain and temperature. It may also occur during aortic procedures. When anterior spinal artery syndromes occur during or after transforaminal epidural steroid injection, the patient may have abrupt back or abdominal pain after injection. An MRI will demonstrate a T2 signal change consistent with cord ischemia/infarct. Anterior spinal artery ischemia may be caused by arteriosclerosis, tumors, thrombosis, hypotension, air or fat embolism, toxins, or other causes. Particulate (steroid) substances, arterial injury, or vascular spasm are other potential causes and have been implicated as significant possibilities for the occurrence of ischemic events after transforaminal epidural steroid injections.
100
435. A57-year-old diabetic male presents with a new onset of neck pain over the past several hours; the pain is beginning to move down each arm equally. Two days ago he had a cervical epidural injection which he receives periodically for a herniated disc. On physical examination, his temperature is 102.4°F, his cervical spine is exquisitely tender to palpation and he complains of radicular pain down both arms. The most likely organism causing this presentation is (A) Pseudomonas (B) Escherichia coli (C) Streptococcus pneumoniae (D) Hemophilus influenza (E) Staphylococcus aureus
435. (E) A spinal epidural abscess must be recognized promptly and treated quickly, otherwise extreme morbidity can result. It may be separate or associated with vertebral osteomyelitis. Diabetic, alcoholic, IV drug using patients, and immunocompromised patients are all at increased risk. Staphylococcus aureus is the most common organism involved. An affected patient usually presents with neck pain that rapidly progresses to radicular symptoms. Quadriplegia can result if left untreated. Treatment involves surgical removal and antibiotic management.
101
436. The following statements are true regarding the pathologic mechanism in HIV-related neuropathy, EXCEPT (A) HIV is found within endoneurial macrophages (B) HIV is found within Schwann cells (C) antisulfatide antibodies are one of the humoral factors responsible for demyelinating diseases in AIDS patients (D) secretion of cytokines by the HIVinfected glial cells may generate tissuespecific autoimmune attack (E) the pathologic mechanisms in HIV-related neuropathies are not well understood
436. (B) The pathophysiology of HIV-related neuropathies is still not well understood. The current understanding is that it is not related to the direct effect of the virus itself. HIV is not found within ganglionic neurons of Schwann cells, but only in endoneurial macrophages, which may generate a tissue-specific autoimmune response by secretion of cytokines, which, in turn, promotes trafficking of activated T cells and macrophages within the endoneurial parenchyma.
102
437. Pain syndromes of neuropathic nature occur in approximately 40% of AIDS patients with pain. Several types of peripheral neuropathies have been described in patients with HIV and AIDS. The most common painful neuropathy encountered in patients with HIV and AIDS is (A) mononeuritis multiplex (B) polyradiculopathy (C) cauda equina syndrome (D) painful toxic neuropathy (E) predominantly sensory neuropathy of AIDS
437. (E) The predominantly sensory neuropathy of AIDS affects up to 30% of people with HIV infection and AIDS and is the most commonly encountered.
103
438. The most important pathophysiologic event in sickle cell anemia, which explains most of its clinical manifestations, is vascular occlusion. The following are the pathophysiologic processes that lead to vascular occlusion in patients with sickle cell disease (SCD), EXCEPT (A) erythrocyte dehydration (B) distortion of the shape of erythrocytes (C) polymerization of the sickle cell hemoglobin on deoxygenation (D) decreased deformability of erythrocytes (E) decreased stickiness of erythrocytes
438. (E) The primary process that leads to vascular occlusion is the polymerization of sickle cell hemoglobin on deoxygenation, which in turn results in distortion of the shape of red blood cells (RBCs), cellular dehydration, decreased deformability, and increased stickiness of RBCs, which promotes their adhesion to and activation of the vascular endothelium.
104
439. Aphysician has to exercise extra caution when attributing SCD patient’s complaints of pain to behavioral deviations, such as drug-seeking behavior, because (A) patients in real pain, such as sickle cell pain, do not develop addiction to opioids (B) most patients with SCD have substance abuse and addiction, as they are exposed to opioids early in life (C) there is a higher incidence of controlledsubstance diversion in SCD patients (D) sickle cell pain could be the prodrome of a serious and potentially fatal complication of SCD (E) severe pain, such as sickle cell pain, should only be managed by an experienced physician subspecializing in pain management
439. (D) SCD is unlike other pain syndromes where the provider can make decisions on treatment based solely on the pain and its associated behavior. A primary care physician, for example, taking care of a middle-aged patient with job-related low back pain may decide to expel the patient from his or her care if the patient in question demonstrates suspicious drug-seeking behavior. Doing the same with patients who have SCD could be counterproductive. There are anecdotes of patients with SCD who were dismissed from certain programs only to be found dead at home within 24 hours after dismissal or to be admitted to other hospitals with serious complications. Sickle cell pain could be the prodrome of a serious and potentially fatal complication of SCD in some patients.
105
440. What makes the pain of SCD unique in its acuteness and severity? (A) SCD patients tend to have a decreased threshold to pain because of prolonged and early exposure to severe pain in life (B) SCD patients have increased tolerance to opioids and opioid-related hyperalgesia (C) SCD pain pathophysiology involves a combination of ischemic tissue damage and secondary inflammatory response (D) Repetitive SCD crises lead to ischemic damage of the CNS and subsequent central sensitization to pain (E) SCD patients tend to anticipate and respond with a spectacular behavioral manifestation to pain, because of its cyclic feature
440. (C) Tissue ischemia caused by vascular occlusion resulting from in situ sickling causes infarctive tissue damage, which in turn initiates a secondary inflammatory response. The secondary response may enhance sympathetic activity by means of interactions with neuroendocrine pathways and trigger release of norepinephrine. In the setting of tissue injury, this release causes more tissue ischemia, creating a vicious cycle. It is the combination of ischemic tissue damage and secondary inflammatory response that makes the pain of SCD unique in its acuteness and severity.
106
441. At initial presentation, objective signs of a painful SCD crisis, such as fever, leukocytosis, joint effusions, and tenderness, occur in (A) less than 10% of patients (B) about 25% of patients (C) about 50% of patients (D) about 75% of patients (E) more than 90% of patients
441. (C) Objective signs of a painful crisis, such as fever, leukocytosis, joint effusions, and tenderness, occur in about 50% of patients at initial presentation.
107
442. What percentage of hospital admissions in adult SCD patients result from acute sickle cell pain? (A) Less than 10% (B) About 25% (C) About 50% (D) About 75% (E) More than 90%
442. (E) Pain is the hallmark of SCD, and the acute sickle cell painful episode (painful crisis) is the most common cause of more than 90% of hospital admissions among adult patients who have SCD.
108
``` 443. Which of the following is true regarding treatment of sickle cell pain with NSAIDs? (A) They should be completely avoided because of potential side effects (B) They should not be administered continuously for more than 5 days (C) They should be administered only in combination with opioids (D) They should not be administered continuously for more than 1 month (E) Potential morbidity from their side effects in SCD patients is the same as in the general population ```
443. (B) NSAIDs have potentially serious, systemic adverse effects. They include gastropathy, nephropathy, and hemostatic defects. It is advisable not to administer them continuously for more than 5 days to patients with SCD.
109
444. Pharmacologic management of SCD pain includes three major classes of compounds: nonopioids, opioids, and adjuvants. Nonopioids include acetaminophen, NSAIDs, topical agents, tramadol, and corticosteroids. The following is true about Tramadol, EXCEPT (A) it inhibits neuronal reuptake of serotonin and norepinephrine (B) it acts as a weak μ-receptor agonist (C) it does not have a “ceiling” effect because of its safe side-effect profile (D) it is not associated with an addiction potential (E) it is a centrally acting analgesic
444. (D) Tramadol is a synthetic, centrally acting analgesic that is not chemically related to opioids. It acts as a weak agonist with preferential affinity to the μ-receptors. Moreover, it inhibits neuronal reuptake of both serotonin and norepinephrine and stimulates the release of serotonin. Thus, it has functional properties of an opioid and an antidepressant. This drug received an initial enthusiastic reception based on the perception that it was not associated with clinically significant respiratory depression or addiction potential. However, this enthusiasm waned after reports indicated that seizures may be an adverse effect and that abuse potential is increasing.
110
445. All of the following are true about chronic pain in the spinal cord injury (SCI) patient, EXCEPT (A) approximately two-thirds of all SCI patients suffer from chronic pain (B) approximately one-third of SCI patients with pain have severe pain (C) pain in SCI patients may lead to severe depression and even suicide (D) because of the overwhelmingly significant impairment of other important functions, pain is only a minor consideration in an SCI patient (E) pain in SCI interferes with rehabilitation and activities of daily living (ADLs)
(D) Chronic pain is a major complication of SCI. Epidemiologic studies indicate that approximately two-thirds of all SCI patients suffer from chronic pain out of which one-third have severe pain. Pain interferes with rehabilitation, daily activities, quality of life, and may have significant influence on mood leading to depression and even suicide.
111
``` 446. In an SCI patient, chronic pain secondary to overuse is common in (A) neck (B) lower back (C) shoulders and arms (D) hips and thighs (E) knees and feet ```
446. (C) Musculoskeletal pain is common in both the acute and chronic phase of SCI. Chronic pain secondary to overuse is common in shoulders and arm, and vertebral column pain may occur because of the secondary changes following fractures and fixation, mechanical instability, and osteoporosis.
112
``` 447. Autonomic dysreflexia usually occurs after an SCI at (A) any level (B) above C4 (C) above C7 (D) above T6 (E) above L1 ```
447. (D) Autonomic dysreflexia may complicate SCI patients with a lesion above the splanchnic outflow (sixth thoracic level).
113
``` 448. The following is true regarding the visceral pain in an SCI patient, EXCEPT (A) it is unlikely that visceral pain may occur in the absence of any abdominal organ dysfunction (B) the pattern of visceral pain is not affected in an SCI patient, because it is transmitted through the sympathetic system, which usually bypasses the site of injury (C) autonomic dysreflexia cannot be triggered by visceral pain (D) visceral pain is always present in an SCI patient as part of the central pain syndrome (E) increases in spasticity or autonomic reactions may be the only indications of abdominal organ dysfunction ```
448. (E) Visceral pain usually presents as dull or cramping abdominal uncomfortable and painful sensations, which may be associated with nausea and autonomic reactions. It is likely that visceral pain may occur in the absence of any abdominal organ dysfunction, and may in some cases represent a neuropathic type of pain. SCI patients may not have the typical signs of abdominal illness, and they should be carefully examined whenever any new pain or changes in existing pain occur. Increases in spasticity, pain at any location, or autonomic reactions may be the only indications of abdominal organ dysfunction.
114
449. Neuropathic pain in SCI is divided into abovelevel, at-level, and below-level types. Depending on the type of pain, nerve root injury (peripheral component), and/or SCI (central component) may contribute to the pain. Which of the following is true? (A) Below-level pain has only peripheral component (B) At-level pain has only peripheral component (C) Below-level pain is usually caused by compressive mononeuropathy (D) Below-level pain is usually caused by CRPS (E) At-level pain may have both peripheral and central components
449. (E) Above-level neuropathic pain includes pain caused by compressive mononeuropathies (particularly carpal tunnel syndrome) and CRPS. While below-level pain is considered to be a central pain caused by the spinal cord trauma, at-level pain may have both peripheral (nerve root) and central (spinal cord) components that are difficult to separate.
115
450. One of the characteristics of stimulus-evoked neuropathic pain in SCI can be temporal summation of pain. Temporal summation of pain is defined as (A) elicitation of pain by nonnoxious stimulation (B) pain continuing after stimulation has ceased (C) an increased pain response to a noxious stimulus (D) abnormal increase in pain with each repetitive stimulation (E) pain felt in a place apart from the stimulated area
450. (D) Answers (A), (B), (C), and (D) define allodynia, aftersensation, hyperalgesia, and referred pain, respectively.
116
451. An axonal injury triggers a Wallerian degeneration, which is defined as (A) degeneration of the portion of the axon separated from the neuronal body by the injury (B) degeneration of the injured neuron distal and proximal to the level of injury (C) atrophy of the motor unit supplied by the injured neuron (D) dying of the body of the neuron, which lost its axon (E) degeneration of the secondary afferent neuron because of the absence of the input from the injured primary afferent neuron
451. (A)
117
452. Which of the following is true about the central cord syndrome? (A) It is the injury of the mid-portion of the spinal cord, usually around T6 level (B) Upper extremities are affected more than lower (C) It is very uncommon (D) Patient usually presents with absent perianal sensation (E) It is usually associated with complete SCI
452. (B) Central cord syndrome is one of the incomplete SCI syndromes. It is the most common pattern of injury, representing central gray matter destruction with preservation of only the peripheral spinal cord structures, the sacral spinothalamic and corticospinal tracts. The patient usually presents as a quadriplegic with perianal sensation and has an early return of bowel and bladder control. Any return of motor function usually begins with the sacral elements (toe flexors, then the extensors), followed by the lumbar elements of the ankle, knee, and hip. Upper extremity functional return is generally minimal and is limited by the degree of central gray matter destruction.
118
453. A common sensory symptom in patients with CRPS is hyperpathia which may be defined as (A) normally innocuous stimuli are perceived as painful (B) exaggerated pain perception after a noxious stimulus at the site of injury (C) exaggerated pain perception after a noxious stimulus in the area surrounding the primary affected skin (D) exaggerated delayed painful perception after a noxious stimulus (E) unpleasant abnormal sensation, whether spontaneous or evoked
453. (D) Sensory symptoms and signs in CRPS include spontaneous pain, hyperpathia, allodynia, and hyperalgesia. Answer (A) is the definition of allodynia, answer (B) defines primary hyperalgesia, and answer (C) is secondary hyperalgesia. Answer (E) is the definition of dysesthesia. Dysesthesia maybe spontaneous.
119
454. Common findings in patients with CRPS I include (A) symmetrical distal extremity pain (B) pain intensity that is usually proportional to the intensity of the initiating event (C) nearly all patients with CRPS I having sweating abnormalities (D) sensory abnormalities that are most often proximal (E) consistency between the inciting lesion and the spatial distribution of the pain
454. (C) CRPS I is a painful condition following an injury, which may not even be a neuropathic pain, as not obvious lesion is present. Patients with this condition develop asymmetrical distal extremity pain, which is disproportionate to the intensity of the initiating event. Sensory abnormalities appear early in the course of the disease and are more pronounced distally. No clear relationship between the injury and the area of pain distribution exist. Sweating abnormalities, whether hypohidrosis or hyperhidrosis are present in nearly all patients with CRPS I.
120
``` 455. Late changes observed in patients with CRPS I include (A) sensory abnormalities (B) warm extremities (C) distal swelling (D) trophic changes (E) increased dermal blood flow ```
455. (D) Trophic changes, particularly abnormal hair growth, fibrosis, decreased dermal blood flow, thin glossy skin, and osteoporosis are more common in long standing cases, while the others described present in acute phases of the disease.
121
456. Which of the following is true in relation to CRPS? (A) Males are more commonly affected than females (B) CRPS II is more common than CRPS I (C) Three bone scan showing unilateral periarticular uptake is mandatory to confirm CRPS diagnosis (D) The diagnosis of CRPS is mainly clinical (E) The mean age group is between 15 and 25 years
456. (D) The diagnosis of CRPS I and II follows the IASP clinical criteria. Bone scintigraphy may be a valuable tool to rule out other conditions. CRPS I is more common than CRPS II and the female to male ratio is from 2:1 to 4:1.
122
457. Which of the following is the diagnostic criteria that differentiates CRPS II from CRPS I? (A) Triple-phase bone scan showing diffuse spotty osteoporosis (B) Weakness of all muscles of the affected extremity (C) Sweating abnormalities (D) Lesion of a peripheral nerve structure is mandatory (E) Paresis
457. (D) The symptoms of CRPS II are similar to those of CRPS I, except that in CRPS II, there must be a lesion of a peripheral nerve structure and subsequent focal deficit are mandatory for the diagnosis.
123
458. Patients with CRPS exhibit significant psychologic findings, including (A) the most common psychiatric comorbidities are anxiety and depression (B) increased incidence of somatization in patients with CRPS than to patients with chronic low back pain (C) psychiatric problems are the cause of CRPS (D) CRPS is a psychogenic condition (E) maladaptive behaviors in CRPS patients indicate the presence of psychopathology
458. (A) The majority of patients with CRPS present with significant psychologic distress, being the most frequent anxiety and depression. Current evidence is against the theory that CRPS is a psychogenic condition. The pain in CRPS is the cause of psychiatric problems and not vice versa. When compared to patients with low back pain, CRPS patients showed a higher frequency of somatization, but other psychologic parameters were similar.
124
459. The Lewis triple response consists of the following, EXCEPT (A) activation and sensitization of cutaneous C fibers elicit local edema (B) reddening of the skin at the site of the stimulus (C) spreading flare (D) local peripheral vasoconstriction mediated by the release of substance P
459. (D) Activation and sensitization of cutaneous C fibers elicit a response consisting of a wheal (local edema), a reddening of the skin at the site of stimulus, and a spreading flare, were the responses described by Lewis (1927). [Lewis T. The blood vessels of the human s
125
460. The diagnosis of myofascial pain syndrome is confirmed when (A) the myofascial trigger point is identified by palpation (B) a patient has a widespread muscle condition (C) a patient is diagnosed with fibromyalgia first (D) regional muscle spasms are noted (E) none of the above
460. (A) Muscle pain tends to be dull, poorly localized, and deep in contrast to the precise location of cutaneous pain. The diagnosis of myofascial pain syndrome is confirmed when the myofascial trigger point is identified by palpation. An active myofascial trigger point is defined as a focus of hyperirritability in a muscle or its fascia that causes the patient pain. B. Myofascial pain syndrome is usually thought of as a regional pain syndrome in contrast to fibromyalgia as a widespread syndrome; however, as many as 45% of patients with chronic myofascial pain syndrome have generalized pain in three or four quadrants. Hence, regional pain syndromes should raise a suspicion of myofascial pain syndrome, but patients with widespread musculoskeletal pain can also have myofascial pain syndrome. C. The American Pain Society showed general agreement with the concept that myofascial pain syndrome exists as an entity distinct from fibromyalgia. D. Systemic palpation differentiates between myofascial
126
461. Four experienced physicians examine a patient. They all identify the same precise localization of trigger points within a muscle. The minimum criteria that must be satisfied in order to distinguish a myofascial trigger point from any other tender area in muscle are (A) a taut band and a tender point in that taut band (B) a local twitch response (C) referred pain (D) reproduction of the person’s symptomatic pain (E) A and C
461. (A) The minimum criteria that must be satisfied in order to distinguish a myofascial trigger point from any other tender area in muscle are a taut band and a tender point in that taut band. The existence of a local twitch response, referred pain, or reproduction of the person’s symptomatic pain increases the certainty and specificity of the diagnosis.
127
462. A23-year-old female is found to have a trigger point in the left trapezius muscle. With regard to electrical characteristics of the trigger point, which of the following is false? (A) A characteristic electrical discharge emanates from the trigger point (B) Spontaneous EMG activity typical of end-plate noise occurring in myofascial trigger points has been further confirmed in a study of young subjects with chronic shoulder and arm pain (C) The sympathetic nervous system does not have a modulating effect on the motor activity of the trigger point (D) End-plate noise without spikes was found at trigger point sites to a significantly greater degree than at end-plate zones outside of trigger points, and not at all in taut band sites outside of an end-plate zone (E) All the above statements are true
462. (C) D. This statement is true. End-plate noise is characteristic of, but not restricted to, the region of the myofascial trigger point. Hence, an objective EMG signature of the trigger point is now available for diagnostic and research purposes.
128
463. Manual therapy is one of the four basic treatment options used for inactivating trigger points. Some practitioners also incorporate the stretch and spray technique. It is therapeutic (A) because like with other soothing sprays, the placebo effect is extremely high (B) because the vapocoolant spray stimulates thermal and tactile A-β skin receptors, thereby inhibiting C fiber and A-δ fiber afferent nociceptive pathways and muscle spasms, myofascial trigger points, and pain when stretching (C) because the vapocoolant is focused specifically on the trigger point (D) because therapists don’t have to be as liberal when passively stretching patients (E) B and D
463. (B) A. There is no reason to believe that the placebo effect is unusually high with this or other soothing sprays. B. The stretch and spray technique combines the use of a vapocoolant spray with passive stretching of the muscle. Application of vapocoolant spray stimulates temperature and touch A-β skin receptors, thereby inhibiting C fiber and A-δ fiber afferent nociceptive pathways and muscle spasms, myofascial trigger points, and pain when stretching C. The patient is positioned comfortably and the muscle involved is sprayed with a vapocoolant spray, and then the muscle is stretched passively. With the muscle in the stretched position, the spray is applied again over the skin overlying the entire muscle, starting at the trigger zone and proceeding in the direction of, and including, the referred pain zone. After, the area is heated with a moist warm pack for 5 to 10 minutes. The patient is encouraged to perform full range of motion exercises with the body part. This technique can be used in physical therapy as a separate modality or following myofascial trigger point injections. D. It is expected that therapists can be more liberal once the spray has been applied owing to less patient discomfort.
129
``` 464. Which of the following is not an effective myofascial release technique? (A) Strumming (B) Perpendicular and oscillating mobilizations (C) Therapeutic ultrasound (D) Connective tissue massage (E) Pétrissage ```
464. (C) Myofascial release techniques and sustained pressure may soften and relax contracted and hardened muscles. The principle of the least possible force is applied, instead of applying high stress to the muscle. Effective myofascial release techniques include strumming. Strumming is when a finger runs across a taut band at the level of the trigger point over the nodules from one side of the muscle to the other. The operator’s fingers pull perpendicularly across the muscle rather than along the length of the fibers. When the nodule of the trigger point is encountered, light pressure is maintained until the operator senses tissue release. Other techniques include perpendicular and oscillating mobilizations, tissue rolling, connective tissue massage, and deep muscle massage consisting of effleurage (stroking massage technique) and pétrissage (kneading massage technique). After superficially passing over the muscles and adjacent muscles, massage therapy can be applied directly to the taut band and trigger points. Exercise and massage helped to reduce the number and intensity of trigger points, but the addition of therapeutic ultrasound did not improve the outcome.
130
``` 465. Trigger points can theoretically be related to underlying articular dysfunction. Joint and muscular dysfunction is closely related and should be considered as a single functional unit. It has been noted that (A) restrictions in joint capsules inhibit function for those muscles overlying the particular joint, but muscle dysfunction does not result in joint capsule restrictions (B) restrictions in joint capsules do not inhibit adjacent muscles, nor does muscular dysfunction result in joint capsule restrictions (C) restrictions in joint capsules inhibit muscle function for those muscles overlying the particular joint. Conversely, muscle dysfunction results in joint capsule restrictions (D) restrictions in joint capsules do not severely limit the overlying muscles; however, muscle dysfunction does regulate joint capsule range of motion (E) muscle dysfunction does not result in joint capsule range of motion ```
465. (C) When treating myofascial pain, the physician must evaluate and, if indicated, treat both soft tissue and joint dysfunction. Restrictions in joint capsules hinder the function of the muscles that overlie the joint, while muscle irregularities result in joint capsule restrictions. Zygapophyseal joints may have pain referral patterns that are analogous to myofascial trigger points. The limited range of motion and weakness that results from this somatic dysfunction that affects muscles and joints can be reversed easily by manual therapy.
131
466. An administrative assistant presents with a number of upper back trigger points. She is scheduled for dry needling. What is the most common indication for this therapeutic modality? (A) Relief of an acute myofascial pain syndrome (B) To identify a myofascial trigger point as the cause of a particular pain (C) To eliminate a trigger point permanently (D) Inactivation of myofascial trigger points to facilitate physical therapy (E) None of the above
466. (D) Answers A., B., and D. are the therapeutic, diagnostic, and adjunctive indications for myofascial trigger point needling, respectively, with D. being the most common appropriate use of this technique. Rarely is dry needling done to eliminate a trigger point permanently, although, this can happen when the myofascial pain syndrome is acute. Inactivation of the myofascial trigger point appears to be the result of the mechanical action of the needle in the trigger point itself, because it also occurs when no medication is used. However, using local anesthetics is more comfortable for the patients and results in longer lasting pain reduction. After pinpointing and manually stabilizing the trigger point in the taut band with the fingers, the needle is quickly passed through the skin and into the trigger zone. Alocal twitch response or a report of referred pain indicates that the trigger zone has been entered. One-tenth to 0.2 mL of local anesthetic can be injected. The needle is pulled back to just below the skin, the angle is changed, and it is once again inserted through the muscle to another trigger zone. In this way a funnel-shaped volume of muscle can be evaluated without withdrawing the needle through the skin. The trigger zone is explored this way until no further local twitch responses are obtained. By this time, the taut band is usually gone and the spontaneous pain of the trigger point has subsided. Experienced patients know when trigger points have been inactivated.
132
467. Randomized, double-blind, controlled studies have shown that adding which of the following medications to local anesthetics increases the pain relief obtained from myofascial trigger point injections? (A) Steroids (B) Ketorolac (C) Vitamin B12 (D) Diphenhydramine (E) None of the above
467. (E) A., B., and C. While direct needling of myofascial trigger points appears to be an effective treatment, there is insufficient evidence that needling therapies have efficacy beyond placebo. These researchers also found no evidence to suggest that the injection of one material was more effective than another. They found no advantage to adding steroids, ketorolac, or vitamin B12 to local anesthetic. Steroids actually have the disadvantage that they are locally myotoxic and that repeated administration can produce all the unwanted side effects associated with steroids. For those who are allergic to local anesthetics, saline or dry needling can be used. D. No studies have been done to confirm or refute that diphenhydramine increases the efficacy of myofascial trigger points.
133
``` 468. Which of the following is not a complication of trigger point injections? (A) Local hemorrhage into muscle (B) Infection (C) Transient nerve block (D) Syncope (E) Torticollis ```
468. (E) Complications of trigger point injections: • Local bleeding into muscle • Local swelling • Painful contraction of a taut band from inadequate myofascial trigger point inactivation (missing the trigger point) • Infection • Perforation of a viscous body, most commonly the lung • Nerve injury from direct trauma by the needle • Transient nerve block • Syncope • Allergic reaction from the anesthetic Torticollis is a contraction, often spasmodic, of the muscles of the neck, chiefly those supplied by the spinal accessory nerve; the head is drawn to one side and usually rotated so that the chin points to the other side. While missing the trigger point during needling can cause a painful contraction of a taut band, torticollis has not been noted.
134
469. Two days after a trigger point injection, a patient presents to your office irate. He claims that the trigger point injection has not helped him whatsoever. Which one of the following is not a likely reason why you are having this problem? (A) You missed the trigger point during needling (B) The patient is not motivated to improve (C) You injected the secondary or satellite trigger point and not the primary trigger point (D) Insufficient muscle stretching in the clinic after the injection (E) Not enough stretching by the patient at home
469. (B) Answers (A), (C), (D), and (E) are all causes of trigger point failure. C. Myofascial adhesions can possibly develop with secondary or “satellite” trigger points in nearby muscles. Trigger points appearing in muscles that are part of a functional unit must be treated together. Muscles that work together as agonists or in opposition as antagonists, constitute a functional muscle unit. For example the trapezius and levator scapula muscles work together as agonists in elevation of the shoulder, but are antagonists in rotation of the scapula, the trapezius rotating the glenoid fossa upward and the levator scapula rotating it downward.
135
470. Mechanical precipitating factors can cause unrelenting musculoskeletal pain. The major mechanical factors that practitioners must consider in treating myofascial pain syndrome include anatomic variations, poor posture, and work-related stress. Of the anatomic variations, which of the following are the most common? (A) Leg length discrepancy and small hemipelvis (B) Short femur syndrome (C) Long great toe syndrome (D) Kyphosis (E) All of the above
470. (A) The most common anatomic variations that constitute mechanical factors precipitating myofascial pain are: leg length discrepancy and small hemipelvis, short upper arm syndrome, and the long second metatarsal syndrome. The leg length disparity syndrome produces a pelvic tilt that results in a cascade of chronic contraction and activation of a chain of muscles in an attempt to straighten the head and level the eyes. The quadratus lumborum and paraspinal muscles contract to correct the deviation of the spine caused by the pelvic tilt. Unwarranted loading perpetuates myofascial trigger points and may result in low back, head, neck, and shoulder pain. Trigger points in these persistently shortened and constantly contracted muscles are not easily inactivated until the muscles are unloaded. The quadratus lumborum is less likely to develop trigger points during the teenage years, and typically, unilateral low back pain is located on the side of the shorter leg because of early shrinking of the ipsilateral annulus fibrosis. In adults, it occurs on the side of the longer leg, caused by later spondylitic changes and quadratus lumborum shortening. A true leg length incongruity can be corrected by placing a heel lift on the shorter leg. The asymmetry caused by a small hemipelvis is corrected by placing an ischial or “butt” lift under the ischial tuberosity. Short upper arms cause forward shoulder roll, pectoral muscle shortening, and abnormal loading of the neck and trunk muscles as the individual attempts to find a comfortable position when seated. Along second metatarsal bone obscures the stable tripod support of the foot produced by the first and second metatarsal bones anteriorly and the heel posteriorly. In contrast, in this condition, weight is carried on a knife-edge from the second metatarsal head to the heel, overstressing the peroneus longus that attaches to the first metatarsal bone. Diagnostic callus formation takes place in the abnormally stressed areas: under the second metatarsal head, and on the medial aspect of the foot at the great toe and first metatarsal head. Correction is accomplished with support under the head of the first metatarsal.
136
``` 471. What nutritional or hormonal factors have repeatedly been found to be low in persons with persistent myofascial pain? (A) Iron (B) Folic acid (C) Vitamin B12 (D) Thyroid hormone (E) All of the above ```
471. (E) A. In women with chronic coldness and myofascial pain, ferritin has been found to be below 65%, largely because of an iron intake that is insufficient to replace menstrual loss. GI blood loss caused by antiinflammatories and parasitic diseases can also cause ferritin to be low. Ferritin represents the tissue bound nonessential iron stores in the body that supply the essential iron for oxygen transport and iron-dependent enzymes. Fifteen to 20 ng/mL is low and anemia is common at levels of 10 ng/mL or less. The association between depleted iron and chronic myofascial pain hints that ironrequiring enzymatic reactions may be limited in these people, which may produce an energy crisis in muscle when it is overloaded and thereby produce metabolic stress. Myofascial trigger points will not easily resolve in such instances, and iron supplementation in patients with chronic myofascial pain syndrome and serum ferritin levels below 30 mg/mL prevents or corrects these symptoms. B. and C. Folic acid and vitamin B12 function not only in erythropoiesis but also in central and peripheral nerve formation. Preliminary studies have shown that 16% of patients with chronic myofascial pain syndrome either were deficient in vitamin B12 or had insufficient levels of vitamin B12, and that 10% had low serum folate levels. D. Hypothyroidism can be suspected in chronic myofascial pain syndrome when coldness, dry skin/hair, constipation, and fatigue are also present. One study, found that under these circumstances (chronic) it occurred in 10% of patients. The myofascial trigger points tend to be more extensive in hypothyrotic patients. Hormone replacement may resolve many myofascial complaints and perpetuate a more permanent healthy state by allowing the implementation of physical therapy and trigger point inactivation. Also common in chronic musculoskeletal pain: low vitamin D, recurrent candida yeast infections, elevated uric acid levels, parasitic infections (especially amebiasis), Lyme disease, osteoarthritis, rheumatoid arthritis, Sjögren syndrome, carpal tunnel syndrome, and peripheral neuropathy secondary to DM. The postlaminectomy syndrome is frequently caused by myofascial trigger points.
137
``` 472. Biologic aberrations seen in most patients with fibromyalgia include all of the following, EXCEPT (A) lowered pain thresholds to pressure induced pain (B) disordered sleep as evidenced by polysomnography (C) increased spinal fluid levels of substance P (D) decreased spinal fluid levels of nerve growth factor (NGF) (E) no physiological or biochemical evidence for central sensitization ```
472. (D) Biologic abnormalities that are detected in most fibromyalgia patients include • Dysfunctional sleep by polysomnography • Physiological or biochemical evidence for central sensitization • Temporal summation or second pain • Lowered thresholds to pressure-induced pain detected by brain imaging • Low levels of the biogenic amines to drive descending inhibition of nociception • Elevated spinal fluid levels of substance P • In primary fibromyalgia only, elevated spinal fluid levels of NGF
138
473. Of the following criteria, which are absolutely necessary for the classification of fibromyalgia syndrome? (A) Widespread pain for at least 3 months (B) Pain sensitivity to 4 kg of digital pressure at a minimum of 11 of 18 anatomically defined tender points (C) Diagnosis after the age of 18 years (D) A and B (E) A, B, and C
473. (D) The American College of Rheumatology sanctioned a study that led to the criteria for diagnosing fibromyalgia: a history of widespread pain for at least 3 months and pain sensitivity to 4 kg of digital pressure at 11 or more of 18 anatomically defined tender points. The criteria displayed sensitivity and specificity of 88.4% and 81.1%, respectively, for patients with fibromyalgia against normal control and disease control subjects with other painful conditions.
139
474. Which of the following is not one of the 18 potential locations for tender points in fibromyalgia? (A) Occiput, at the suboccipital muscle insertion (B) Low cervical, at the anterior aspects of the intertransverse spaces at C5-C7 (C) Lumbar paraspinal musculature, from the level of L3 to L5 (D) Lateral epicondyle, extensor muscle, 2 cm distal to the epicondyle (E) Knees, at the medial fat pad proximal to the joint line and condyle
474. (C) In addition to the four areas mentioned in the question stem, five other locations exist: 1. Trapezius, at the middle of the upper muscle border 2. Supraspinatus, near the origins, above the spine of the scapula 3. Second rib, upper surface just lateral to the second costochondral junction 4. Gluteal, in upper outer quadrants of buttocks in anterior fold of muscle 5. Greater trochanter, posterior to the trochanteric prominence
140
475. A 60-year-old female recently diagnosed with fibromyalgia has been exhibiting signs of anxiety and depression and is now in treatment. This situation (A) makes perfect sense because fibromyalgia is a psychogenic disorder (B) relates to the fact that a subgroup of fibromyalgia patients concurrently have depression and anxiety, although an affective disorder is unlikely to cause fibromyalgia (C) demonstrates a patient who is less likely to exhibit signs of depression than one who has not sought out medical care (D) exhibits a patient who was probably abused as a child (E) none of the above
475. (B) A. and B. Fibromyalgia is no longer considered a psychogenic disorder, however, there is a subgroup of fibromyalgia patients with associated depression or anxiety. C. It is believed that fibromyalgia patients who have had medical treatment are more apt to exhibit symptoms of depression than those in the community who have not. D. Sexual abuse in childhood is no longer considered a legitimate hypothesis for the origination of fibromyalgia.
141
``` 476. Which of the following does not occur often as a clinical manifestation of the fibromyalgia syndrome? (A) Irritable bladder syndrome (B) Irritable bowel syndrome (C) Urinary urgency (D) Dizziness and light-headedness (E) A and D ```
476. (E) The female urethral syndrome or irritable bladder syndrome constitutes urinary frequency, dysuria, suprapubic discomfort, and urethral pain despite sterile urine.
142
477. A 50-year-old female with fibromyalgia complains of trouble sleeping. You are not surprised as it is well-known that these patients (A) awaken in the morning feeling stiff, cognitively sluggish, and unrefreshed by their sleep (B) commonly awaken feeling distressingly alert after only a few hours of sleep (mid insomnia) and then are unable to sleep soundly again until near morning (terminal insomnia) (C) don’t have trouble napping during the day (D) A and B (E) A, B, and C
477. (E) A., B., and C. Most patients (90%) with fibromyalgia have trouble sleeping. Some have difficulty getting to sleep (initial insomnia), while the majority awaken feeling alarmingly alert after only a few hours of sleep (mid insomnia) and are then unable to sleep soundly again until near morning (terminal insomnia). They usually awaken in the morning feeling incredibly stiff (lasts 45 minutes to 4 hours), mentally listless, and unrefreshed by their sleep. Hence, it is surprising that they have difficulty napping during the day. Moldofsky (2002) observed that 60% of fibromyalgia patients exhibit an electroencephalogram (EEG) pattern of sleep architecture called alpha wave intrusions of deep, delta wave, non–rapid eye movement sleep, which relates to subjective fatigue and psychologic distress but is not specific for fibromyalgia. Its prevalence in the healthy general population or in those with insomnia or dysthymia is only 25%. [
143
478. Which one of the following statements is false regarding fatigue in fibromyalgia? (A) It should always be attributed to the fibromyalgia itself (B) It is rarely induced by medications (C) It manifests as a feeling of weakness as opposed to the feeling of tiredness felt in chronic fatigue syndrome (D) A and B (E) A, B, and C
478. (E) Approximately 80% of patients with fibromyalgia have fatigue, while a small percentage of these actually meet the criteria for chronic fatigue syndrome (CFS). CFS is thought to affect approximately 4 per 1000 adults. For unknown reasons, CFS occurs more often in women and in adults in their 40s and 50s. The illness is estimated to be less prevalent in children and adolescents, but study results vary as to the degree. CFS often manifests with widespread myalgia and arthralgia, cognitive difficulties, chronic mental and physical exhaustion, often severe, and other characteristic symptoms in a previously healthy and active person. There remains no assay or pathologic finding which is widely accepted to be diagnostic of CFS. It remains a diagnosis of exclusion based largely on patient history and symptomatic criteria, although a number of tests can aid diagnosis. The fatigue of CFS is a feeling of weakness, while the fatigue of fibromyalgia is a feeling of tiredness. Fatigue may result from sedating medications (ie, TCAs being used for the treatment of insomnia in fibromyalgia). The rest of the differential diagnosis is quite extensive and must take into account: • Sleep disorders • Chronic infections • Autoimmune disorders • Psychiatric comorbidities • Neoplasia
144
479. Secondary fibromyalgia refers to (A) fibromyalgia that does not interfere with a patient’s functioning (B) fibromyalgia that occurs in the setting of another painful condition or inflammatory disorder (C) a multifocal pain syndrome that occurs only after a patient has been diagnosed with dysthymia (D) fibromyalgia that meets all the other characteristics of the disease but only produces between 8 and 10 tender points (E) none of the above
479. (B) The rest of the answers are blatantly wrong. Secondary fibromyalgia may not be clinically distinguishable from that of primary fibromyalgia. Examples of secondary fibromyalgia: • Rheumatoid arthritis patients have fibromyalgia 30% of the time • Systemic lupus erythematosus (SLE) 40% • Sjögren syndrome 50% • Lyme disease 20%; the symptoms of fibromyalgia may develop 1 to 4 months after infection, often in association with Lyme arthritis. The signs of Lyme disease will normally resolve with antibiotics, but the fibromyalgia symptoms can persist • Chronic hepatitis • Inflammatory bowel disease • Tuberculosis • Chronic syphilis • Bacterial endocarditis • AIDS • Hypothyroidism • Hypopituitarism • Hemochromatosis Patients with rheumatic disease and concomitant fibromyalgia experience joint pain out of proportion to their synovitis. The practitioner should treat each of the conditions separately, because increasing the dosage of antirheumatic medications in the absence of active inflammation might have minimal effect on the pain augmented by the fibromyalgia.
145
480. Avery inquisitive 40-year-old female, recently diagnosed with fibromyalgia, states that she has been reading about her condition on the Internet. She wants to know about substance P. You tell her that (A) substance P is a pronociceptive neurochemical mediator of pain because it carries or amplifies afferent signals (B) substance P levels in the patients with fibromyalgia have been found to be significantly higher in the CSF, serum, and urine (C) the elevation of substance P in the CSF is a result of lowered CSF substance P esterase (D) the elevation in CSF substance P is indicative of fibromyalgia (E) A and B
480. (A) A. There are several neurochemical mediators of pain that appear to be factors in the pathogenesis of fibromyalgia: • Substance P • NGF (elevated in primary fibromyalgia, but not secondary) • Dynorphin A (normal or elevated in fibromyalgia) • Glutamate • Nitric oxide • Serotonin (decreased in fibromyalgia) • Noradrenaline (its inactive metabolite is significantly lowered in fibromyalgia) Substance P, NGF, dynorphin A, glutamate, and nitric oxide are considered pronociceptive because they transmit or intensify afferent signals, leading to the brain perceiving increased pain. On the other hand, serotonin, noradrenaline, the amino terminal peptide fragment of substance P, and endogenous opioids are considered to be antinociceptive because they hinder the transmission of nociceptive signals. B. All studies on substance P in fibromyalgia patients have found significantly higher average concentrations (two- to threefold) of substance P than in the CSF of healthy control subjects. However, the levels in other bodily fluids like saliva, serum, and urine, have been normal in fibromyalgia. C. The increased substance P is not because of decreased CSF substance P esterase activity, because the rate of cleavage of labeled substance P was found to be normal. In primary fibromyalgia, it is believed that NGF may be responsible for the elevated CSF substance P through its effects on central sensitization and neuroplasticity. D. Increased CSF substance P is not specific to fibromyalgia as it is also seen in painful rheumatic diseases irrespective of whether they have fibromyalgia. In patients that were status post–total hip replacement, elevated substance P prior to the procedure normalized after the surgery when the pain was gone. Certain chronic conditions such as low back pain and diabetic neuropathy present with lower than normal CSF substance P levels.
146
481. The management objectives for fibromyalgia are (A) not specific because there is still no cure (B) reestablish emotional balance (C) improve sleep (D) restore physical function (E) all of the above
``` 481. (E) There is no current cure for fibromyalgia, so its management is • Nonspecific • Multimodal • Expectant • Symptomatic The goals are to • Decrease pain • Enhance sleep • Reinstate physical function • Maintain social interaction • Restore emotional balance • Decrease the excessive use of health care resources The best way to achieve these goals is through a multidisciplinary approach of • Education • Exercise • Physical therapy • Medications • Social support ```
147
482. The shared decision concept (A) is a method where half of the treatment decisions come from the physician and half come from the ancillary staff (physical therapists, massage therapists, etc) (B) improves both patient and physician satisfaction (C) must be used on selective patients because cultural background, beliefs, and religion can all inhibit its effectiveness (D) A and B (E) A, B, and C
482. (B) The shared decision concept emphasizes the importance of simultaneous exchange of information until an agreement between the doctor and patient can be achieved concerning available diagnostic and treatment approaches. It improves both patient and physician satisfaction, is preferred by patients, and sets the stage for better outcomes. A. The shared decision concept involves the physician and the patient. C. The physician may outline the treatment options with associated risks and benefits, while the patient may disclose information about their culture, fears, expectations, beliefs, and attitudes.
148
483. A 60-year-old female recently diagnosed with fibromyalgia wants to discuss her treatment options. She is adamant about not taking medications. Which one of the following statements is false regarding her alternatives? (A) Relaxation techniques like progressive muscle relaxation, self-hypnosis, or biofeedback have been recommended (B) Cognitive behavioral therapies and support groups are efficacious is some patients (C) Aerobic exercise can yield positive outcomes (D) Heat and cold applications can provide relief (E) Deep massage does more harm than good
483. (E) A. All these techniques have been recommended for some patients with fibromyalgia. Progressive muscle relaxation was developed by Jacobson, who argued that since muscular tension accompanies anxiety, one can reduce anxiety by learning how to relax the muscular tension. Jacobson trained his patients to voluntarily relax certain muscles in their body in order to reduce anxiety symptoms. He also found that the relaxation procedure is effective against ulcers, insomnia, and hypertension. Selfhypnosis is a naturally occurring state of mind which can be defined as a heightened state of focused concentration (trance), with the willingness to follow instructions (suggestibility). Biofeedback is a form of alternative medicine that involves measuring a subject’s quantifiable bodily functions such as blood pressure, heart rate, skin temperature, sweat gland activity, and muscle tension, conveying the information to the patient in real time. This raises the patient’s awareness and conscious control of their unconscious physiological activities. By providing the user access to physiologic information about which he or she is generally unaware, biofeedback allows users to gain control of physical processes previously considered an automatic response of the autonomous nervous system. B. Cognitive-behavioral therapies have improved pain scores, pain coping, pain behavior, depression, and physical functioning over several months in fibromyalgia patients. It is suspected that follow-ups with booster sessions may prolong the effects. While some think that support groups perpetuate griping, a resource-oriented selfsupport group can help a fibromyalgia patient come to terms with an illness and provide invaluable patient education. C. Aerobic exercise is one of the first nonpharmacologic strategies promoted for patients with fibromyalgia. Low-impact aerobics of sufficient intensity to produce cardiovascular stimulation can decrease pain, enhance sleep, improve mood, increase energy, advance cognition, and better a patient’s overall outlook. Fibromyalgia patients who exercise deal better with the disease. However, a fibromyalgia patient can also experience increased pain if the exercise regimen is too strenuous or carried out during an inopportune time in the treatment. These patients should begin with low-impact exercises (ie, aqua therapy). Continuing the patient on the exercise regimen becomes easier as the patient’s pain decreases. D. Heat helps fibromyalgia patients with tenderness, stiffness, and cephalgia. It can also calm muscles, ease exercising, and accentuate a sense of well-being. Cold application also works. E. Some patients do obtain relief by light massages that progress to more deep sedative ones.
149
484. A PhD student comes with questions. She wants to know how dopamine and serotonin play a role in fibromyalgia pathogenesis. You tell her that (A) dopamine levels directly correlate with pain levels (B) tryptophan, serotonin, 5-hydroxytryptophan, and 5-hydroxyindole acetic acid have been found to be decreased in fibromyalgia patients (C) the number of tender points in fibromyalgia patients have not been found to correlate with the concentration of serotonin in the serum (D) dopamine agonists have been found to decrease pain in fibromyalgia patients (E) B and D
484. (E) A. and D. Dopamine is a neurotransmitter best known for its role in the pathology of schizophrenia, Parkinson disease, and addiction. There is also strong evidence for a role of dopamine in restless leg syndrome, which is a common comorbid condition in patients with fibromyalgia. In addition, dopamine plays a critical role in pain perception and natural analgesia. Accordingly, musculoskeletal pain complaints are common among patients with Parkinson disease, which is characterized by drastic reductions in dopamine owing to neurodegeneration of dopamine-producing neurons, while patients with schizophrenia, which is thought to arise, at least partly, from hyperactivity of dopamine-producing neurons, have been shown to be relatively insensitive to pain. Interestingly, patients with restless legs syndrome have also been demonstrated to have hyperalgesia to static mechanical stimulation. Fibromyalgia has been commonly referred to as a “stress-related disorder” owing to its frequent onset and worsening of symptoms in the context of stressful events. It was therefore proposed that fibromyalgia may represent a condition characterized by low levels of central dopamine that likely results from a combination of genetic factors and exposure to environmental stressors, including psychosocial distress, physical trauma, systemic viral infections, or inflammatory disorders (eg, rheumatoid arthritis, systemic lupus erythematosus). This conclusion was based on three key observations: (1) fibromyalgia is associated with stress; (2) chronic exposure to stress results in a disruption of dopamine-related neurotransmission; and (3) dopamine plays a critical role in modulating pain perception and central analgesia in such areas as the basal ganglia including the nucleus accumbens, insular cortex, anterior cingulate cortex, thalamus, periaqueductal gray, and spinal cord. In support of the “dopamine hypothesis of fibromyalgia,” a reduction in dopamine synthesis has been reported after using positron emission tomography (PET) and demonstrated a reduction in dopamine synthesis among fibromyalgia patients in several brain regions in which dopamine plays a role in inhibiting pain perception, including the mesencephalon, thalamus, insular cortex, and anterior cingulate cortex. A subsequent PET study demonstrated that, whereas healthy individuals release dopamine into the caudate nucleus and putamen during a tonic experimental pain stimulus (ie, hypertonic saline infusion into a muscle bed), fibromyalgia patients fail to release dopamine in response to pain and, in some cases, actually have a reduction in dopamine levels during painful stimulation. Moreover, a substantial subset of fibromyalgia patients respond well in controlled trials to pramipexole, a dopamine agonist that selectively stimulates dopamine D2/D3 receptors and is used to treat both Parkinson disease and restless legs syndrome. B. Tryptophan is decreased in the serum and CSF of fibromyalgia patients. Serotonin is low in fibromyalgia serum. 5-Hydroxytrytophan, the intermediary between tryptophan and serotonin, and 5-hydroxyindole acetic acid, the by-product of serotonin metabolism, are both low in the CSF of patients with fibromyalgia. The excretion in urine of 5-hydroxyindole acetic acid was lower than normal in patients with fibromyalgia, lower in females versus males, and lower in females with fibromyalgia versus females who don’t have fibromyalgia. C. The numbers of active tender points in fibromyalgia patients directly correlated with the concentration of serotonin in fibromyalgia sera.
150
485. No treatments seem to be working for a 45- year-old female with a 5-year history of fibromyalgia. She has talked to a relative who told her that oxycodone/acetaminophen works for all pain. How do you respond? (A) Opioids work but only when a shortacting medication is combined with a long-acting medication (B) In combination with pregabalin and duloxetine, hydromorphone has displayed incredible synergy in extremely depressed fibromyalgia patients (C) Women with fibromyalgia have reduced μ-opioid receptor availability within regions of the brain that normally process and dampen pain signals (D) Fibromyalgia is so difficult to treat that you are willing to try anything that she thinks may help (E) You are not opposed to trying opioids, but the potential hyperalgesia has been found to be significantly worse in patients with fibromyalgia
485. (C) Fibromyalgia, a common chronic pain condition characterized by widespread pain, is thought to originate largely from altered central neurotransmission. In this study, a sample of 17 fibromyalgia patients and 17 age- and sexmatched healthy controls, were compared using μ-opioid receptor PET. PET scans measure blood flow in the brain. It was demonstrated that fibromyalgia patients display reduced μ-opioid receptor binding potential within several regions known to play a role in pain modulation, including the nucleus accumbens, the amygdala, and the dorsal cingulate. The reduced availability of the receptors could result from a reduced number of opioid receptors, enhanced release of opioids that are produced naturally by the body, or both. These findings indicate altered endogenous opioid analgesic activity in fibromyalgia and suggest a possible reason for why exogenous opiates appear to have reduced efficacy in this population. The reduced availability of the receptor was associated with greater pain among people with fibromyalgia. Answers (A), (B), (D), and (E) have no merit.
151
486. Numerous medications have been used to treat the insomnia associated with fibromyalgia. Which one of the following has not been used? (A) Amitriptyline (B) Cyclobenzaprine (C) Fluoxetine (D) Clonazepam (E) Pregabalin
486. (C) A. and B. The sedating tricyclic biogenic amine reuptake drugs, such as amitriptyline and cyclobenzaprine, are the most commonly prescribed medications for fibromyalgia insomnia. These medications are mostly used in low doses to improve sleep and to enhance the effects of analgesics (amitriptyline 10-25 mg at night and cyclobenzaprine 5-10 mg at night). Patients can develop tachyphylaxis to them, but a 1-month holiday from the drugs may help restore effectiveness. C. SSRIs are so stimulating that they can interfere with sleep, and should never be taken at bedtime. D. Benzodiazepine decrease anxiety and allow less troubled sleep (alprazolam, clonazepam). Clonazepam in particular can help control nocturnal myoclonus when it is associated with fibromyalgia. E. Pregabalin is a sedative in addition to an antinociceptive medication.
152
``` 487. Pathophysiologic components of cancer pain can be (1) somatic (nociceptive) pain (2) sympathetic pain (3) neuropathic pain (4) central pain ```
487. (E)
153
``` 488. The skeletal sites most commonly involved in osteolytic metastatic processes are (1) ribs (2) humerus (3) femur (4) tibia ```
488. (B) In osteolytic bone metastases, the most commonly involved sites are vertebrae, pelvis, ribs, femur, and skull. Upper and lower extremity bones, except femur, are not commonly involved.
154
489. The primary compression of the spinal cord from metastatic deposits occurs in (1) the thoracic spine in 70% of patients (2) the lumbar spine in 20% of patients (3) the cervical spine in 10% of patients (4) multiple sites of the spine in 60% of patients
489. (A) Multiple sites of metastatic epidural spinal cord compression occur in 17% to 30% of all patients. This is particularly common in prostatic and breast carcinoma and uncommon in lung cancer.
155
490. In a patient with skeletal metastases, bisphosphonates (1) inhibit recruitment and function of osteoclasts (2) stimulate osteoblasts (3) have greatest effect in breast cancer and multiple myeloma (4) have an acute pain-relieving effect
490. (E) Bisphosphonates decrease resorption of bone directly, by inhibiting the recruitment and function of osteoclasts, and indirectly, by stimulating osteoblasts. In patients with bony metastases, they are the standard therapy for hypercalcemia after rehydration, and have the greatest effect in patients with breast cancer and multiple myeloma. Bisphosphonates also have an acute pain-relieving effect, which is thought to be derived from the reduction of various pain-producing substances.
156
491. The following substance(s) may be useful in treating a patient with a malignant disease: (1) Gabapentin (2) Amitriptyline (3) Samarium 153 (4) Hydromorphone
491. (E) Both gabapentin, an antiepileptic drug, and amitriptyline, TCA, are widely used in treating neuropathic pain, which is often a significant component of a cancer pain syndrome. Samarium 153 belongs to the group of bone-seeking radiopharmaceuticals emitting medium- to high-energy beta particle radiation. The most commonly used agent in this group is Strontium 89 with a documented pain-relieving effect in patients with bony metastases. Samarium 153, rhenium 186, and phosphorus 32 are also available for clinical use.
157
492. The following is (are) the possible compilation( s) of a neurolytic celiac plexus block: (1) Persistent diarrhea (2) Aortic pseudoaneurysm (3) Intradiscal injection (4) Damage to the artery of Adamkiewicz
492. E)
158
493. In a cancer pain patient, the following agent(s) can be used effectively via implantable intrathecal delivery system: (1) Opioids (2) α2-Adrenergic agonists (3) Local anesthetics (4) Ziconotide
493. (E) Both gabapentin (an antiepileptic drug) and amitriptyline (a TCA) are widely used in treating neuropathic pain, which is often a significant component of a cancer pain syndrome. Samarium 153 belongs to the group of bone-seeking radiopharmaceuticals emitting medium- to high-energy beta particle radiation. The most commonly used agent in this group is Strontium 89 with a documented pain-relieving effect in patients with bony metastases. Samarium 153, rhenium 186, and phosphorus 32 are also available for clinical use.
159
494. Which of the following conditions are the possible complications of chemotherapy in a cancer patient? (1) Toxic peripheral neuropathy (2) PHN (3) Avascular necrosis (4) Pseudorheumatism
494. (E)
160
495. As compared to younger subjects which of the following is correct about older people with pain? (1) There may be difficulties in determining the etiology of pain in older people (2) Older people generally receive significantly lower amounts of opioid analgesia (3) There may be increased potency of opioids (4) The majority of older people choose quantity of life over quality of life
495. (E)
161
``` 496. The goals of palliative care can be summarized as follows: (1) To help those who need not die to live, and to live with the maximum of freedom from constraints on their quality of life arising from acute and chronic conditions of the body (2) To help those who can no longer live to die on time—not too early and not late (3) To help the dying, whether in hospital, nursing home, hospice, or at home, to die with dignity and in peace (4) To administer euthanasia only to the patients who truly understand the fact that their condition is terminal and who personally request it ```
496. (A)
162
``` 497. Peripheral neuropathy(ies) is (are) characterized by (1) sensory loss (2) fasciculations (3) dysesthesias (4) chronic pain ```
497. (B) In clinical practice, most peripheral neuropathies do not produce chronic pain as impairment of nerve fibers carrying nociception should result in decrease pain perception. In most neuropathies, all components of the peripheral nervous system are affected, presenting with variable sensorimotor deficit and autonomic dysfunction.
163
498. Area(s) of acute pain processing in cortical and subcortical regions of the brain as determined by functional MRI include (1) anterior cingulate cortex (2) parietal cortex (3) prefrontal cortex (4) hypothalamus
498. (B) The most commonly activated areas during acute processing of pain in humans are S-I, S-II, anterior cingulated cortex, insular cortex, prefrontal cortex, thalamus, and cerebellum.
164
499. Small-diameter peripheral neuropathies are commonly painful. Example(s) of these neuropathies include (1) Ross syndrome (segmental anhidrosis) (2) Fabry disease (3) Charcot-Marie-Tooth disease type 1 (4) diabetic neuropathy
499. (C) Many studies suggest that axonal injury along the nociceptive fiber in the peripheral nervous system is the main cause of neuropathic pain. Several conditions where the small fibers are spared support this concept. The Charcot-Marie-Tooth disease also known as hereditary motor and sensory neuropathy where the demyelination is limited to large myelinated fibers, do not manifest with pain. Segmental anhidrosis or Ross syndrome where only autonomic fibers are affected, is also not painful. On the other hand, conditions affecting the small nerve fibers, like diabetic neuropathy or Fabry disease, a rare lipid-storage disorder, commonly present with pain.
165
``` 500. Chronic renal failure neuropathy is commonly manifested with (1) restless leg syndrome (2) painful neuropathy (3) distal weakness (4) selective loss of small nerve fibers ```
500. (A) Chronic renal failure is associated with selective loss of large nerve fibers which is rarely painful. Common symptoms include restless leg syndrome, distal numbness, and paresthesias, with distal weakness usually in the lower extremities.
166
``` 501. Animal studies in neuropathic pain conditions have shown (1) intraplantar injections of interleukin 1(IL-1) reduces mechanical nociceptive threshold (2) IL-1 hyperalgesia is mediated by bradykinin B-1 receptors (3) effects of IL-1 on mechanical hyperalgesia seems to be mediated by prostaglandins (4) IL-1 effects on nociceptions may be mediated by vagal afferents ```
501. (E) Cytokines, a heterogeneous group of peptides activate the immune system and mediate inflammation. They form a complex bidirectional system that communicates between the immune system and the CNS. IL-1 is the most extensively studied cytokine. Intraplantar as well as intraperitoneal injections of IL-1 reduce mechanical and probably thermal nociceptive threshold, which may be blocked by local cyclooxygenase inhibitors, supporting the role of prostaglandins in the process. The communicating pathway between the peripheral cytokines and the brain may involve vagal afferents terminating in the nucleus tractus solitarius and circumventricular sites that lack a blood-brain barrier.
167
502. Potential complication(s) of stellate ganglion block include (1) pneumothorax (2) lesion of the recurrent laryngeal nerve (3) neuritis (4) Horner syndrome
502. (A) Blockade of the sympathetic innervation of the head can be documented by the presence of Horner syndrome, which is characterized by myosis, ptosis, and enophthalmus. Associate findings include conjunctival injection, nasal congestion, and facial anhidrosis. Horner syndrome is an expected finding after blockade of the sympathetic afferents to the face and can not be considered a complication.
168
503. Important factor(s) involved in the development of neuropathic pain include (1) behavioral studies have shown that NMDA is involved in the induction and maintenance of pain-related behaviors (2) the spinal N-type voltage-dependent calcium channels are the predominant isoform involved in the pre- and postsynaptic processing of sensory nociceptive information (3) tactile allodynia in the spinal nerve ligation model may be blocked by intrathecal N-type Ca2+ blockers like ziconotide (4) after nerve injury there is upregulation of the NMDA receptors
503. (E) Behavioral studies have shown that activation of NMDA receptors are required for the development and maintenance of pain-related behaviors. Calcium channels are the key ion involved in the release of transmitters. Different subtypes of calcium channels (L-, N-, and P/Qtypes) may have a differential role depending on the nature of the pain state. The N-type voltagedependent calcium channels appear to be the predominant isoform involved in the pre- and postsynaptic processing of sensory nociceptive inputs. Animal and clinical studies have shown partial pain relief with the use of a specific N-type calcium channel blocker synthetically derived from a conotoxin, SNX-111. The generic name of this substance derived from the snail’s natural conotoxin is Ziconotide.
169
504. Which of the following is (are) effect(s) of μ-opioid agonists in neuropathic pain conditions? (1) Decrease dynamic allodynia (2) Decrease temperature threshold for cold pain (3) Decrease static allodynia (4) μ-Opioid agonists do not have any beneficial effects in patients with neuropathic pain conditions
504. (A) The effectiveness of opioid agonists in the management of neuropathic pain has created significant controversy over the last two decades. Recent studies have increased our understanding about this topic. In patients with SCI and stroke, IV morphine showed poor effects in reducing spontaneous pain, but significantly reduced stroking allodynia. Other studies used alfentanil in the treatment of neuropathic pain independently of the etiology and observed decrease in dynamic, stroking allodynia, and spontaneous pain, while increase the temperature at which heat pain was detected and decrease the temperature at which cold pain was detected.
170
505. Effect(s) of GABA in the modulation of afferent nociceptive input include (1) GABAA produces postsynaptic inhibition via metabotropic receptors, which are ligand-gated Cl− channels (2) the dominant type of inhibition of glutaminergic excitatory postsynaptic action potential is produce by GABA and/or glycine (3) GABAB and adenosine produce postsynaptic hyperpolarization by activation of K+ channels (4) GABA and glycine produce slow activation of postsynaptic potentials
505. (A) Action potentials in the dorsal horn neurons are mediated by glutaminergic excitatory postsynaptic potentials, this activity may be inhibited predominantly by the inhibition produced by GABA and/or glycine which causes fast inhibition of postsynaptic potentials. GABAAand glycine receptors are ligand-gated Cl− channels, while GABAB, adenosine, and opioids exert their typically produced postsynaptic hyperpolarization by activation of K+ channels.
171
506. During the windup process (1) sustained depolarization may recruit K+ channels, leading to decrease in the intracellular Ca2+ levels (2) cumulative recruitment of NMDAreceptor current leads to progressive relief of the Mg2+ blockade of the NMDA-receptor pore (3) more intense or sustained noxious peripheral stimulation induces a decrease in the release of neuromodulator peptides, leading to an excitatory state (4) intracellular calcium levels play a major role in the development of windup
506. (C) More intense or sustained noxious peripheral stimulation induce primary afferent nociceptors to discharge at higher frequencies and to release from central nociceptor terminals neuromodulator peptides like CGRP, substance P, and glutamate. As more and more dorsal horn neurons get depolarized, NMDA receptors open by removing the Mg2+ blockade, allowing for intracellular calcium levels to increase. The end result of these intracellular signaling cascades is windup.
172
507. In patients with PHN (1) histopathologic studies in patients with PHN commonly show ganglion cell loss and fibrosis (2) sensory loss function in the affected dermatome with increase heat pain perception is an almost universal finding (3) antiviral drugs used in chronic PHN usually are ineffective in alleviating pain (4) cold stimuli–evoked pain is more common than heat-evoked pain
507. (B) Histopathological studies in patients with PHN have found ganglion cell loss and fibrosis and atrophy of the dorsal horn, DRG, dorsal root, and peripheral nerves. Up to 30% of patients with PHN have no loss of sensation in the affected dermatome demonstrating minimal or no loss of neuronal function and interestingly thermal sensory thresholds are not affected or even decreased. Antiviral drugs have shown disappointing results in patients with chronic PHN. Heat hyperalgesia is more common than cold hyperalgesia, which only occurs in less than 10% of the patients.
173
508. The Special Olympics has brought together thousands of people with disabilities. Often enough, the race times in these events are significantly better than those of participants in the traditional games. In addition to undergoing the same type of grueling training regimens, however, participants in the Special Olympics often have to deal with difficulties performing activities of daily living as well as comorbidities associated with their primary disease. The likelihood that an amputee in a wheelchair race has phantom limb pain would be decreased if the (1) participant is a young child (2) participant is a male (3) participant is a congenital amputee (4) amputation is a below the knee amputation versus an above the knee amputation
508. (B) Phantom pain is equally frequent in men and women and is not influenced by age in adults, side or level of amputation, and cause (civilian versus traumatic) of amputation. Phantom pain is less frequent in young children and congenital amputees.
174
509. A triple amputee (bilateral lower extremities, left upper extremity) presents to the pain clinic for work-up and treatment of phantom limb pain. This patient’s pain most likely (1) occurs intermittently (2) is primarily localized to the fingers or palm of the hand in the upper extremity or toes, feet, or ankles in the bilateral lower extremities (3) is of stabbing, shooting, or pins and needles character (4) presents with attacks that last several minutes to an hour
509. (A) Phantom pain is usually intermittent. Only a few patients are in constant pain. Episodes of pain are most often reported to occur daily, or at daily or weekly intervals. Phantom pain is usually localized to the distal parts of the missing limb. Pain is normally felt in the fingers and palm of the hand in upper limb amputees and toes, foot, or ankle in lower limb amputees. This may be because of the larger cortical representation of the hand and foot as opposed to the lesser representation of the more proximal parts of the limb. The character of phantom pain is usually described as shooting, pricking, burning, stabbing, pins and needles, tingling, throbbing, cramping, and crushing
175
510. True statement(s) about phantom sensations is (are): (1) They are less frequent than phantom pains (2) They usually appear 1 month after the amputation (3) The phantom sensation usually manifests as enlargement of the missing limb (4) A common position of the phantom for upper limb amputees is the fingers clenched in a fist
510. (D) Phantom sensations are more common than phantom pain, and are experienced by nearly all amputees. The incidence of phantom sensations ranges from 71% to 90%, 8 days to 2 years after amputation. Duration and frequency, but not incidence, decrease as time passes. Phantom sensations are less common in congenital amputees and in patients who underwent amputation before the age of 6 years Nonpainful sensations normally appear within the first days after amputation. The amputee often wakes up from anesthesia with a feeling that the amputated limb is still there. Just after the amputation, the phantom limb often resembles the preamputation limb in shape, length, and volume. As time passes, the phantom fades, leaving back the distal parts of the limb. For example, upper limb amputees may feel hand and fingers, and lower limb amputees may feel foot and toes. Commonly, upper limb amputees feel the fingers clenched in a fist, while the phantom limb of lower limb amputees is often described as toes flexed In some cases, phantom sensations include feeling of movement and posture; however, in others only suggestions of the phantom are felt. Telescoping (shrinkage of the phantom) is reported to occur in about one-third of patients. The phantom progressively approaches the stump and eventually becomes attached to it. It has even been experienced within the residual limb. Phantom pain does not retard shrinkage of the phantom.
176
511. Which of the following is (are) example (s) of how the peripheral nervous system may play a role in phantom pain modulation? (1) Dorsal root ganglion (DRG) cells display an altered expression pattern of different sodium channels (2) Generation, but not maintenance of phantom pain by the sympathetic nervous system (3) Long after limb amputation, injection of noradrenaline around a stump neuroma is reported to be intensely painful (4) Phantom pain is directly related to the skin temperature of the stump
511. (B) Changes occur in the DRG cells, after a complete nerve cut. Cells in the DRG show similar abnormal spontaneous activity and increased sensitivity to mechanical and neurochemical stimulation. Local anesthesia of neuromas abolished tap-induced afferent discharges and tapinduced accentuation of phantom pain, but spontaneous pain and recorded spontaneous activity were unchanged which is consistent with the activity in DRG cells. DRG cells exhibit major changes in the expression of sodium channels, with an altered expression pattern of different channels. The sympathetic nervous system may play a role in generating and especially in maintaining phantom pain. It was noted that application of noradrenaline or activation of the postganglionic sympathetic fibers excites and sensitizes damaged, but not normal nerve fibers. Injecting noradrenaline into the skin can reestablish neuropathic pain that has just been relieved with a sympathetic block and injecting into a neuroma is reported to be intensely painful. The catecholamine sensitivity may also manifest itself in the skin with a colder limb on the amputated side, and it has been implied that phantom pain intensity is inversely related to the skin temperature of the stump.
177
512. Phantom pains are often a replica of preamputation pain. It has also been noted that amputees with phantom pain have more often suffered from intense and long-lasting preamputation pain than have patients without phantom pain. These observations led to the premise that preemptive analgesia may help decrease postamputation pain. Of the studies done on this subject matter (1) most have been of very poor methodological quality (2) the two which included blinding and randomization showed no significant differences versus controls (3) the aim has been to thwart spinal sensitization by blocking the cascade of intraneuronal responses that take place after peripheral nerve injury (4) the sample size was always greater than 100
512. (A) 1. Numerous studies on preemptive analgesia using epidural, epidural/perineural, and just perineural administration have been conducted. Only two were noted to utilize proper patient randomization and blinding. 2. Persistent pain has been reported in up to 80% of patients after limb amputation. The mechanisms are not fully understood, but nerve injury during amputation is important, with evidence for the crucial involvement of the spinal NMDA receptor in central changes. The study objective was to assess the effect of preemptively modulating sensory input with epidural ketamine (an NMDA antagonist) on postamputation pain and sensory processing. 3. The aim of preemptive analgesia is to avoid spinal sensitization by blocking, in advance, the cascade of intra-neuronal responses that take place after peripheral nerve injury. True preemptive treatment is not likely possible in patients scheduled for amputation as most have been suffering from ischemic pain and are almost certainly presenting with preexisting neuronal hyperexcitability. To conclude: epidural blockade has been shown to be effective in the treatment of preoperative ischemic pain and postoperative stump pain.
178
513. Which of the following dietary modification(s) should be made to alleviate symptoms of interstitial cystitis? (1) Restrict spicy foods (2) Eliminate alcohol intake (3) Cease smoking (4) Increase orange juice intake
513. (A) Acidic foods, such as orange juice, carbonated drinks, tomatoes, and vinegar may aggravate the symptoms of interstitial cystitis. Spicy food, alcohol, coffee, chocolate, tea, cola, and smoking should also be either restricted or completely eliminated.
179
514. Sodium pentosan polysulfate (Elmiron) (1) is an antispasmodic medication (2) is an oral analogue of heparin (3) alleviates symptoms of interstitial cystitis by relaxing smooth musculature (4) increases antiadherent surface of the bladder lining
514. (C) Sodium pentosan polysulfate (Elmiron) is an oral analogue of heparin. Inside the bladder, it acts as a synthetic glycosaminoglycan layer and fortifies bladder wall defenses from bacteria by increasing the antiadherent surface of the bladder mucosa.
180
515. Which of the following substance(s) is (are) thought to be involved in descending inhibition? (1) GABA (2) Serotonin (3) Endogenous opioid peptides (4) Norepinephrine
515. (E) Endogenous opioid peptides as other neurotransmitters, such as serotonin, norepinephrine, and GABA, are thought to be involved in descending inhibition.
181
``` 516. Which of the following is (are) the psychological factor(s) affecting pain response? (1) Fear and helplessness (2) Sleep deprivation (3) Anxiety (4) Cultural differences ```
516. (E) Anxiety, fear, helplessness, and sleep deprivation are part of the vicious cycle of pain. Cultural background has been shown to play a significant role in the individual’s pain response.
182
517. Which of the following is (are) psychological method(s) for reducing pain? (1) Placebo and expectation (2) Psychological support (3) Procedural and instructional information (4) Cognitive coping strategies
517. (E)
183
``` 518. Gastrointestinal (GI) impairment in a postsurgical patient can be (1) worsened by increased sympathetic activity because of severe pain (2) contributed to by administration of opioids (3) mitigated by early mobilization of the patient (4) worsened by epidural blockade with local anesthetic ```
518. (A) There is evidence that pain-related impairment of intestinal motility may be relieved by epidural local anesthetics.
184
519. Which of the following feature(s) suggest neuropathic pain? (1) Pain in the area of sensory loss (2) Good response to opioids (3) Pain in response to nonpainful stimuli (4) Absence of Tinel sign
519. (B) Pain in the area of sensory loss, also called deafferentation pain, or anesthesia dolorosa, is a prominent sign of a neuropathic pain. Neuropathic pain generally responds less well to opioid treatment than somatic pain. One of the significant signs of neuropathic pain is also allodynia-a painful response to nonpainful stimuli. Tapping of neuromas produces radiating electric shock sensation in the distribution of the damaged nerve is called Tinel sign— another feature of a neuropathic pain.
185
520. The correct corresponding vertebral levels for optimal epidural catheter placement for various surgical procedures are (1) T10-12 for lower abdominal surgery (2) T8-10 for upper abdominal surgery (3) L2-4 for lower extremity surgery (4) C7-T2 for upper extremity surgery
520. (E)
186
521. Which of the following medications are useful in an inpatient management of a post–burn injury pain? (1) Opioids (2) Ketamine (3) Benzodiazepines (4) Nitrous oxide
521. (E) Post–burn injury pain has two components: a constant background pain and an intermittent procedure-related pain. Continuous infusion of opioids is useful for control of the background component of pain. Pain related to wound care, dressing changes, and others (procedure-related pain) may require brief but profound analgesia. This may be achieved by administration of supplemental IV opioids, or addition of the adjuvant drugs, such as IV ketamine, IV benzodiazepines, inhaled nitrous oxide-oxygen mixture, or even general anesthesia.
187
522. In patients with a traumatic chest injuries thoracic epidural analgesia has been shown to significantly improve inspiratory effort, negative inspiratory force, gas exchange, ability to cough, and ability to clear bronchial secretions. The following finding may be considered relative contraindications for epidural analgesia in a patient with posttraumatic chest injury: (1) Inadequate coagulation function (2) Spine fractures (3) Inadequate intravascular volume resuscitation (4) Concomitant head injury
522. (E) All answers are correct. Aconcomitant head injury may be associated with increased intracranial pressure, which could be considered a contraindication for an epidural catheter placement.6yt
188
523. The most common characteristics of pain in PHN include (1) steady burning or aching (2) dull and poorly localized (3) paroxysmal and lancinating (4) usually not aggravated by contact with the affected skin
523. (B) Two types of pain may be found in PHN: a steady burning or aching, the other, a paroxysmal, lancinating pain. Both may occur spontaneously and are usually aggravated by any contact with the involved skin.
189
524. Which of the following group(s) of medications was found to be useful in treatment of PHN? (1) Opioids (2) Antiepileptic drugs (3) Topical agents (4) Antidepressants
524. (E) All of the listed groups of medication were found to be effective to some extent in the treatment of PHN. Amultimodal approach seems to be more effective because of the synergistic effect different modes of action
190
525. Which of the following is (are) true about interventional therapy for PHN? (1) No proven surgical cure for PHN has been found (2) Cryotherapy is likely to bring only short-term relief (3) Topical lidocaine may provide effective analgesia for PHN (4) Transcutaneous nerve stimulation (TENS) has been shown to give an effective symptomatic relief in some patients
525. (E)
191
``` 526. Antiviral agents in the acute phase of herpes zoster (1) competitively inhibit DNA polymerase, terminating DNA synthesis and viral replication (2) are generally well tolerated (3) hasten healing of the rash (4) may reduce the duration of PHN ```
526. (E)
192
527. Oral steroids for acute herpes zoster (1) are not currently recommended (2) may provide pain relief in the acute phase (3) have no benefit in prevention of PHN (4) have almost no side effects in patients with herpes zoster
527. (A) Oral steroids may provide pain relief in acute phase of herpes zoster, as well as shorten the time to full crusting of lesions. However, controlled trials showed no benefit in the prevention of PHN. With the development of antiviral agents, the use of oral steroids is currently not recommended because of more frequent side effects
193
528. Which of the following statement(s) is (are) true about diabetic amyotrophy? (1) It is commonly associated with pain (2) It responds well to a complicated multimodal treatment (3) Involves weakness and atrophy of the involved muscles (4) Sciatic nerve and its supplied muscles are most commonly affected
528. (B) Diabetic amyotrophy is a condition occurring in diabetic patients, more commonly with type-2 diabetes, which begins with pain in the thighs, hips, and buttocks. Weakness and atrophy of the proximal pelvic muscles groups, iliopsoas, obturator, and adductor muscles follows the painful manifestations. It usually does not involve sciatic nerve, or distal muscles of the lower extremity. The therapy for diabetic amyotrophy is primarily supportive.
194
529. Charcot joint (1) affects primarily weight-bearing joints (2) can be caused by multiple causes other than DM (3) is related to the destruction of afferent proprioceptive fibers (4) is related to the destruction of efferent neural fibers
529. (A) Neuropathic arthropathy (Charcot joint) develops most often in weight-bearing joints. The predominant cause is DM, but also associated with neuropathic arthropathy are leprosy, yaws, congenital insensitivity to pain, spina bifida, myelomeningocele, syringomyelia, acrodystrophic neuropathy, amyloid neuropathy, peripheral neuropathy secondary to alcoholism and avitaminosis, SCI, peripheral nerve injury, postrenal transplant arthropathy, intraarticular steroid injections, and syphilis. The etiology of Charcot joint is believed to be related to the destruction of afferent proprioceptive fibers and subsequent unrecognized trauma to the joint.
195
530. Which of the following statement(s) about treatment of diabetic peripheral neuropathic pain (DPNP) is (are) true? (1) Most of the antidepressants are Food and Drug Administration (FDA) approved for the treatment of DPNP (2) Most therapies for DPNP result in more than 90% reduction in pain (3) Most of the anticonvulsant drugs are FDA approved for the treatment of DPNP (4) NSAIDs are the most commonly utilized medications
530. (D) Across-sectional, community-based survey of 255 patients with DPNP recruited through the offices of endocrinologists, neurologists, anesthesiologists, and primary care physicians found that NSAIDs were the most commonly used medications, with 46.7% reporting their use. This is despite the fact, that there is little evidence to support the efficacy of NSAIDs in DPNP, and that NSAIDs have a high potential for renal impairment in patients with diabetic neuropathy.
196
531. Treatment of painful diabetic neuropathy (PDN) rests on modification of the underlying disease and control of pain symptoms. In turn, the modification of the underlying disease includes strict glycemic control. Which of the following is (are) true? (1) Tight glycemic control can halt or slow the progression of distal sensorimotor neuropathy (2) Hemoglobin A1c target should be
531. (E)
197
532. The current treatments of the PDN include (1) antiepileptic drugs (2) antidepressants (3) opioids (4) aldose reductase inhibitors
532. (A) Given the frequency of imperfect glycemic control, attempts have been made to identify oral medications which can target downstream metabolic consequences of hyperglycemia, thereby preventing production of reactive oxygen species, which are felt to contribute to diabetic neuropathy. Unfortunately, trials of aldose reductase inhibitors, which decrease aberrant metabolic flux, have been disappointing (eg, sorbinil, zopolrestat).
198
533. The convulsive tic (1) is more severe in males (2) may indicate the presence of a tumor, vascular malformation, or ecstatic dilation of the basilar artery (3) is because of presence of bilateral facial spasms (4) is a result of painful periodic unilateral facial contractions
533. (B) The combination of trigeminal neuralgia and hemifacial spasm is known as convulsive tic. It is reported to be more severe in women than in men. Occasionally, strong spasms involve all of the facial muscles unilaterally almost continuously. Seldom, facial weakness may be present. Convulsive tic may indicate the presence of a tumor, vascular malformation, or ecstatic dilation of the basilar artery, compressing the trigeminal or facial nerves.
199
534. Which of the following support(s) the diagnosis of idiopathic trigeminal neuralgia? (1) Periods of weeks or months without pain (2) Increase pain by commonly benign stimuli, like talking, eating, or washing (3) Pain often alleviated by sleep (4) Bilateral pain in the distribution of the trigeminal nerve, described as shooting or lancinating
534. (A) The diagnostic criteria for trigeminal neuralgia are: shooting, electric-like, sharp, severe pain which last for seconds but sometimes experienced together with pain-free intervals. The pain is periodic with weeks or months without pain. The pain is typically unilateral and triggered by light touch, eating, talking, or washing.
200
535. Which of the following is (are) true regarding trigeminal neuralgia? (1) Very often, trigeminal neuralgia is the presenting symptom in patients affected with multiple sclerosis (2) Trigeminal neuralgia is 20 times more common in patients with multiple sclerosis (3) Trigeminal neuralgia tends to occur in the early stages of multiple sclerosis (4) Bilateral trigeminal neuralgia is seen more often than expected in patients with multiple sclerosis
535. (D) Rarely, trigeminal neuralgia is the presenting symptom of multiple sclerosis. More often, trigeminal neuralgia presents in patients with advanced stages of multiple sclerosis.
201
536. The retrogasserian glycerol injection (1) is a selective neurolytic agent with preference for sensory fibers, leaving intact motor neurons (2) recurrence rates are the highest of all ablative techniques (3) sensory loss is almost unseen in patients after this procedure (4) sensory loss is less common than with radiofrequency thermocoagulation
536. (C) Glycerol is a nonselective neurolytic agent. Although less common than with radiofrequency thermocoagulation, the frequency of patients affected by sensory loss is high. Recurrence rate is the highest among all ablative techniques.
202
537. Which of the following is (are) true for trigeminal neuralgia? (1) Trigeminal neuralgia is the most common cranial neuralgia (2) It is more common in females (3) The highest incidence is in elderly patients (4) The disease most frequently linked with trigeminal neuralgia is multiple sclerosis
537. (E) The trigeminal neuralgia is the most common cranial neuralgia and its most frequent form is idiopathic. The incidence of trigeminal neuralgia is 5.7 per 1000 females and 2.5 per 1000 in males. Patients with multiple sclerosis have a higher risk for trigeminal neuralgia. Other potential relation was found in patients with family history.
203
538. Potential factors involved in the development of trigeminal neuralgia include (1) ion channel upregulation in the area of the trigeminal injury (2) focal demyelination (3) up to 30% of patients with trigeminal neuralgia have arterial cross compression at the level root entry zone (4) cell body degeneration in the trigeminal complex of the mesencephalon
538. (A) The trigeminal neuralgia is a primary axonal degenerative disease. The ignition hypothesis combines the current knowledge of the role of ion channels in the development of neuropathic pain. Focal demyelination adjacent to the area of arterial compression has been shown by electron microscopy in patients undergoing posterior fossa surgery. In up to 30% of patients with arterial cross compression, there is a groove or an area of discoloration lateral where the root entry zone would be expected.
204
539. Spontaneous intracranial hypotension (SIH) (1) the most common site of idiopathic dural tears is the lower lumbar region (2) congenital subarachnoid or Tarlov cysts are a potential site for dural weakness and rupture (3) the most obvious difference between PDPH and SIH, is the lack of postural symptoms in the second (4) are no characteristic findings on MRI
539. (C) SIH and PDPH have similar presentation, pathophysiology, and treatment. The most important distinction is the initiating event, which is obvious in PDPH. MRI of the brain with gadolinium enhancement in patients with SIH shows meningeal enhancement, and thickening, and a possible caudad shift of the brain toward the foramen magnum. The most common location of spontaneous dural tear is the thoracic region followed by the cervicothoracic and thoracolumbar junction regions.
205
540. A common treatment for patients with PDPH is epidural blood patch (EBP). Which of the following is (are) true regarding this therapy? (1) Maintenance of supine position for 2 hours after the patch provides higher chances for success (2) As a result of the predominant caudad spread of the blood after EBP, a level of placement above the suspected dural tear is recommended (3) The effectiveness of EBP is reduced when the dural tear was caused by a large-size needle (4) The long-term relief of an initial EBP is close to 98%
540. (B) It has been shown that keeping a supine position for 2 hours after the EBP provides higher chances of success when compared with 30 minutes. Although initial relief is very high (close to 100%), the overall long-term relief of PDPH after EBP is between 61% and 75%. The effectiveness of EBP is reduced when the dural tear was caused by a large-size needle.
206
541. The incidence of PDPH is between 1% and 75%. Factor(s) that prevent its development at the time of dural puncture include (1) use of an interlaminar approach (2) use of intrathecal catheter (3) bed rest after the puncture (4) use of small-gauge spinal needle
541. (C) Two important factors in the prevention of PDPH are the use of small and blunt bevel spinal needles. Other factors that may prevent the development of PDPH include the use of paramedian approach (with angles of 35° or greater) and the use of intrathecal catheters. Bed rest as a preventive measure is not effective.
207
542. Diagnostic criteria for cervicogenic headache by the International Headache Society and the International Association for the Study of Pain (IASP) include (1) unilateral headache (2) relief of acute attacks by blocking the greater occipital nerve with local anesthetic (3) aggravation of the headache with neck movements (4) decrease range of neck motion
542. (E) Cervicogenic headache is defined as headache that arises from painful disorders of structures in the upper neck, which generates irritation of the upper cervical roots or their nerve branches. The current classification by the IHS and the IASP accepts these headaches to be unilateral or bilateral. All the other options in the questions are true.
208
543. The cervicogenic headache (1) has a prevalence of 0.4% to 2.5% in the general population and may account for up to 15% to 20% of patients with chronic headache (2) is more common in females; a female to male ratio of 4 to 1 (3) mean age is the beginning of the fourth decade (4) is aggravated by neck movement, and alleviated by occipital nerve block
543. (E) According to the International Headache Society and the IASP, cervicogenic headache (CGH) is a pain originated in the neck, mostly unilateral although it may be bilateral, exacerbated by neck movement, and alleviated by local anesthetic block of the occipital nerve. The prevalence of CGH is 0.4% to 2.5% in the general population and may account for up to 15% to 20% of patients with chronic headache, more common in females with a 4:1 ratio and mean age of patients with 42.9 years.
209
544. The cortical spreading depression (1) may produce the aura symptoms (2) produces activation of the trigeminal nerve endings (3) consist of decreased cerebral blood flow spreading forward from the occipital cortex (4) is followed by generalized cerebral vascular dilation that explains the headache
544. (A) During the aura in classic migraine a decrease in cerebral blood flow decreases spreads from the occipital cortex. The variation in the cerebral blood flow causes the aura and activates trigeminal nerve endings. It is possible that cortical spreading depression may stimulate peripheral nerve terminals of the nucleus caudalis trigeminalis.
210
545. In terms of migraine which of the following is (are) true? (1) Migraine with aura is associated with an increase of cerebral blood flow that happens after the headache begins (2) In migraine with aura there is a decrease of cerebral blood flow that starts after the headache begins (3) In migraine without aura there is no change in cerebral blood flow (4) In migraine without aura there is increase of cerebral blood flow before the headache begins
545. (B) During the aura in classic migraine there is a decrease in cerebral blood flow. In patients with classic migraine (migraine with aura), there is increase of cerebral blood flow that happens after the headache begins and this change persist until the headache resolves. In migraine without aura, there is no change in cerebral blood flow.
211
546. Migraine is a risk factor for (1) major depression (2) manic episodes (3) anxiety disorders (4) panic disorders
546. (E) Migraine is a risk factor for affective disorders. When comparing with nonmigraineurs, patients with migraine have a 4.5-fold increased risk of major depression, a sixfold risk of manic episodes, a threefold increase in anxiety disorder, and a sixfold prevalence in panic disorder.
212
547. Migraine happens in 18% of women, 6% of men, and 6% of children. Migraine usually (1) begins in the first three decades of life (2) is of higher prevalence in the fifth decade (3) decrease symptoms in the last trimester of their pregnancy in most females (4) is improved, common after surgical menopause
547. (A) Although migraine begins in the first three decades of life, the higher prevalence is in the fifth one. Family history is a common finding, and pregnant females often experience worsening symptoms in the first trimester and improvement during the third. Many women experience improvement of their symptoms after natural, but not surgical menopause.
213
548. Tension-type headache (1) is the result of sustained contraction of the pericranial muscles with subsequent ischemic pain (2) has more common onset during adolescence and young adulthood (3) has increased EMG activity in muscles with tenderness (4) reduces CNS levels of serotonin that may be responsible for abnormal pain modulation
548. (C) Muscle tenderness is common in patients suffering from TTH, but is not secondary to pericranial muscle contraction or ischemic pain in response to emotion or stress. Increased EMG activity is independent of tenderness and pain. Reduced pain threshold observed in chronic TTH may be the result of low CNS levels of serotonin. Although TTH can begin at any age, the most common onset is during adolescence and young adulthood. The prevalence of TTH decreases with increasing age.
214
549. For any structure to be deemed a cause of low back pain, it must have the following characteristic(s): (1) A nerve supply (2) Be capable of causing low back pain in healthy volunteers (3) Be susceptible to disease or injuries known to be painful (4) Be shown to be a source of pain in a patient using diagnostic techniques of known reliability and validity
549. (E) For any structure to be considered as a source for low back pain it must have the following characteristics: a nerve supply, the capability of causing low back pain similar to what is seen clinically (ideally in healthy volunteers), a susceptibility to disease or injuries known to be painful, and should be able to be shown as a source of pain using diagnostic techniques of known reliability and validity
215
550. Randomized controlled trials (RCTs) have generated evidence-based conclusions for preventive interventions back and neck pain. Which of the following statement(s) is (are) true based on the evidence of RCTs? (1) Lumbar supports are not effective in preventing neck and back pain (2) Exercise may be effective in preventing neck and back pain (3) Back schools are not effective in preventing back and neck pain (4) Ergonomic interventions are effective in preventing back and neck pain
550. (A) There have been RCTs showing that lumbar supports and back schools are not effective in preventing back pain. Exercise has been proven by RCTs to prevent back pain. To date, there are no RCTs on the effectiveness of ergonomics in preventing back pain.
216
551. Anomalies of lumbar nerve roots include which of the following? (1) Two pairs of nerve roots arise from a single dural sleeve (2) A dural sleeve arises from a lower position in the dural sac (3) The vertebral foramen is unoccupied by a nerve or contains a supernumerary set of roots (4) Extradural anastomoses between roots in which a bundle of nerve fibers leaves one dural sleeve to enter an adjacent one
551. (E) The most significant anomalies of the lumbar nerve roots are aberrant courses and anastomoses between nerve roots. Type 1 anomalies are aberrant courses of which there are two kinds. Type 1A describes two pairs of nerve roots arising from a single dural sleeve, whereas type 1B defines a dural sleeve arising from a low position on the dural sac. Type 2 anomalies include those in which the number of roots in the intervertebral foramen varies. An empty foramen is classified as type 2A, and a foramen with extra nerve roots is known as a type 2B. Type 3 anomalies are those involving extradural anastomoses between roots in which a bundle of nerves leaves on dural sleeve to enter one nearby. Type 3 anomalies may coexist with type 2 anomalies.
217
552. Low back pain is defined as pain perceived within a region bounded (1) superiorly by an imaginary line through the T12 spinous process (2) inferiorly by a transverse line through the posterior sacrococcygeal joints (3) laterally by the lateral borders of the erector spinae (4) within the region overlying the sacrum
552. (E) The IASP published standardized terms to define low back pain as pain perceived to arise for lumbar spinal pain and/or sacral spinal pain. Lumbar spinal pain is defined as pain perceived to arise from the region bordered superiorly by an imaginary line through the T12 spinous process, inferiorly by a line through the S1 spinous process, and laterally by the lateral borders of the erector spinae. Sacral spinal pain is that defined as pain perceived to arise from the region bordered laterally by imaginary vertical lines through the posterior superior and posterior inferior iliac spines, superiorly by a transverse line through the S1 spinous process, and inferiorly by a transverse line through the posterior sacrococcygeal joints.
218
``` 553. Complications of cervical transforaminal injections include which of the following? (1) Cerebellar infarction (2) Cerebral infarction (3) Spinal cord infarction (4) Anterior spinal artery syndrome ```
553. (E) Transforaminal injections have been the cause of some of the most worrisome recent complications. These included cerebellar and cerebral infarct, SCI, and infarction, massive cerebral edema, paraplegia, visual defects with occlusion following particulate depo-corticosteroids, anterior spinal artery syndrome, persistent neurologic deficits, transient quadriplegia, cauda equina syndrome, subdural hematoma, and paraplegia following intracordal injection during attempted epidural anesthesia under general anesthesia.
219
554. Discographic stimulation (formally known as discography) is considered positive if (1) adjacent disc stimulation causes pain (2) thermal stimulation with a wire electrode causes pain (3) pain is reproduced at pressures greater than 80 psi (4) pain is reproduced at pressures less than 50 psi and preferably less than 15 psi
554. (C) Despite its controversial history, disc stimulation (formerly known as discography) remains the only means by which to determine whether or not a disc is painful. The test is positive if upon stimulating a disc the patient’s pain is reproduced provided that stimulation of adjacent discs does not reproduce their pain. Discs are also considered to be symptomatic only if pain is reproduced at injection pressures less than 50 psi and preferably less than 15 psi. At injection pressures greater than 80 psi, some discs are painful in normal individuals. The stimulation of discs has been complemented by another approach, heating a wire electrode that has been inserted into a disc annulus. Heating a disc evokes pain that is perceived in the back. This pain may also radiate to the lower extremities and be responsible for referred pain in the thigh and leg.
220
555. The use of chemonucleolysis for lumbar disc herniations is indicated for which of the following? (1) Contained disc protrusions (2) Extruded disc herniations (3) Herniations unresponsive to nonsurgical management (4) Sequestered disc herniations
555. (B) Chemonucleolysis is indicated for contained disc protrusions causing sciatic pain that have been unresponsive to conservative management. The injection is contraindicated for extruded and sequestered disc herniations, and in patients with cauda equine syndrome. Relative contraindications include previous chymopapain injections, previous surgery for lumbar disc herniation, spinal stenosis, severe degenerative disc or facet osteoarthritis, and spondylolisthesis.
221
556. Causes of FBSS include which of the following? (1) Inappropriate selection of patients (2) Irreversible neural injury (3) Inadequate surgery (4) New injury to nerves and spine
556. (E) Inappropriate or premature selection of patients for surgery is the most common cause of FBSS. The second most common cause is persistence of pain secondary to irreversible neural injury. A less common cause is inadequate surgery. Lastly, a variant of FBSS results from new pathologic processes initiated by the initial surgery.
222
``` 557. Selection criteria for elective lumbosacral spine surgery include (1) radicular pain with corresponding dermatomal segmental sensory loss (2) abnormal imaging study showing nerve root compression (3) signs of segmental instability consistent with symptoms (4) success of conservative therapy ```
557. (A) The American Association of Neurological Surgeons and the American Academy of Orthopedic Surgeons have published criteria for patient selection for elective lumbosacral spine surgery. They are applicable to new patients, as well as FBSS patients. They include the following: 1. Failure of conservative therapy. 2. An abnormal diagnostic imaging study showing nerve root or cauda equina compression and/or signs of segmental instability consistent with the patient’s signs/symptoms. Radicular pain with one or more of the following: (a) corresponding dermatomal segmental sensory loss, (b) corresponding dermatomal motor loss, (c) abnormal deep tendon reflexes consistent with appropriate dermatomes.
223
``` 558. The main types of cervical involvement in rheumatoid arthritis include (1) atlantoaxial subluxation (2) cranial settling (3) subaxial subluxation (4) occipital condyle fractures ```
558. (A) There are three main types of cervical spine involvement in rheumatoid arthritis: atlantoaxial subluxation, cranial settling, and subaxial subluxation. The inflammatory changes affecting synovial joints and bursae target structures lined with a synovial membrane in the cervical spine. Patients with cervical spine involvement are thought to have a more severe form of rheumatoid arthritis, and their prognosis is usually worse. Occipital condylar fractures result from a full-energy blunt trauma complemented with axial compression, lateral bending, or rotational injury to the alar ligament.
224
559. Whiplash and whiplash-associated disorders (WAD) comprise a range of injuries to the neck caused by or related to a sudden distortion of the neck. Characteristics include (1) spinal cord injury (SCI) (2) referred shoulder pain (3) sensory deficits (4) headaches
559. (E) The Québec Task Force (QTF) was a task force sponsored by the Société de l’assurance automobile du Québec, the public auto insurer in Quebec, Canada. In 1995, the QTF submitted a report on WADs which made specific recommendations on prevention, diagnosis, and treatment of WAD. These recommendations have become the base for Guideline on the Management of Claims Involving Whiplash- Associated, a guide to classifying WAD and guidelines on managing the disorder. The full report titled Redefining “Whiplash” was published in the April 15, 1995 issue of Spine. An update was published in January 2001. Four grades of WAD were defined by the QTF : Grade 1: Complaints of neck pain, stiffness or tenderness only but no physical signs are noted by the examining physician. Grade 2: Neck complaints and the examining physician finds decreased range of motion and point tenderness in the neck. Grade 3: Decreased range of motion plus neurologic signs, such as decreased deep tendon reflexes, weakness, insomnia, and sensory deficits. Grade 4: Neck complaints and fracture or dislocation, or injury to the spinal cord.
225
560. Distraction testing allows an examiner to identify neurologic and mechanical abnormalities in the cervical spine. It is characterized by (1) relief of neck pain (2) lifting head from the chin and occiput (3) relief of pressure on zygapophyseal joints (4) examiner standing in front of a standing patient
560. (A) The distraction test is performed with an examiner standing behind a seated patient, lifting their head from the chin and occiput, and removing the weight of the head from the neck. If relief of neck pain occurs, the test might point to foraminal intrusion on a nerve root as the source of pain.
226
561. A 45-year-old male with complaints of cervical neck pain radiating down his left arm is examined by a physician. With one particular maneuver, his pain is exactly reproduced. The test(s) that can reproduce his symptoms include (1) distraction testing (2) Valsalva test (3) spurling maneuver (4) Adson test
561. (A) Spurling maneuver is used to identify nerve root compression or irritation. The head is tilted toward the affected side and manual pressure is applied to the top of the head. Radicular pain should be reproduced with this maneuver. Valsalva test allows a patient to experience painful or sensory changes when bearing down. The test increases intrathecal pressure and exacerbates compression within the cervical canal caused by tumors, infections, disc herniations, or osteophyte changes. The distraction test is performed with an examiner standing behind a seated patient, lifting their head from the chin and occiput, and removing the weight of the head from the neck. If relief of neck pain occurs, the test might point to foraminal intrusion on a nerve root as the source of pain. Adson test is used to assess vascular compromise because of subclavian artery impingement from thoracic outlet syndrome.
227
562. The following are true about HIV infection– related neuropathies, EXCEPT (1) inflammatory demyelinating polyneuropathies occur early in the course of HIV infection (2) vasculitis-related neuropathies occur midcourse in HIV infection (3) distal sensory neuropathies occur late in HIV infection (4) HIV-related neuropathies tend to be nonspecific to the stage of HIV infection
562. (D) Peripheral neuropathic pain syndromes in patients with HIV infection tend to be specific to the stage of HIV infection as outlined in answers (1) through (3).
228
563. Which of the following is (are) true about the predominantly sensory neuropathy of AIDS? (1) The predominant symptom is pain in the soles of the feet (2) Ankle jerks are often absent or reduced (3) As symptoms of the neuropathy progress, they usually remain confined to the feet (4) EMG demonstrate sensory, but not motor involvement
563. (A) In patients with predominantly sensory neuropathy of AIDS, the complaints are mostly sensory. However, the NCV and EMG studies demonstrate both sensory and motor involvement.
229
564. Which of the following group(s) of medication( s) is (are) useful in treatment of pain in HIV and AIDS patients? (1) Opioids (2) Anticonvulsants (3) Psychostimulants (4) Antidepressants
564. (E) Psychostimulants, such as dextroamphetamine, methylphenidate, may be useful agents in patients with HIV infection or AIDS who are cognitively impaired. Psychostimulants enhance the analgesic effects of the opioid drugs. They are also useful in diminishing sedation secondary to opioids. In addition, psychostimulants improve appetite, promote sense of well-being, and improve feelings of weakness and fatigue in patients with malignancies.
230
565. Pathophysiologic tissue injury in SCD generates multiple pain mediators. The facilitators of the pain transmission include (1) bradykinin (2) serotonin (3) substance P (4) dynorphin
565. (B) Tissue injury generates several major pain mediators, including, but not limited to IL-1, bradykinin, K+, H+, histamine, substance P, and CGRP. The pathway for painful stimuli is subject not only to activators, sensitizers, and facilitators but also to inhibitors. Serotonin, enkephalin, β-endorphin, and dynorphin are endogenous central pain inhibitors.
231
``` 566. Which of the following established four components of SCD is responsible/associated with the patient’s pain? (1) Anemia and its sequelae (2) Organ failure (3) Comorbid conditions (4) Pain syndromes ```
566. (E) SCD is a quadrumvirate of: (1) pain syndromes, (2) anemia and its sequelae, (3) organ failure, including infection, and (4) comorbid conditions. Pain, however, is the insignia of SCD and dominates its clinical picture throughout the life of the patients. Pain may precipitate or be itself precipitated by the other three components of the quadrumvirate.
232
567. Which of the following statement(s) is (are) true about avascular necrosis (AVN) and SCD? (1) Core decompression is an effective treatment of late stages of the AVN (2) Treatment of AVN is mostly symptomatic (3) AVN affects mostly femoral head (4) AVN is the most common complication of SCD in adults
567. (C) Avascular necrosis is the most commonly observed complication of SCD in adults. Although it tends to be most severe and disabling in the hip area, it is a generalized bone disorder in that the femoral and humeral heads and the vertebral bodies may be equally affected. Treatment of avascular necrosis is symptomatic and includes providing nonopioid or opioid analgesics in the early stages of the illness; advanced forms of the disease require total joint replacement. Core decompression appears to be effective in the management of avascular necrosis if performed during its early stages
233
``` 568. Which of the following statement(s) is (are) true about leg ulceration and SCD? (1) Leg ulcers occur in 5% to 10% of the adult SCD patients (2) Skin grafting is a very effective treatment for chronic leg ulcers in SCD patients (3) Many leg ulcers heal within a few months with good localized treatment (4) Regranex, used to treat leg ulcers, contains an autologous platelet-derived growth factor ```
568. (B) Leg ulceration is a painful and sometimes disabling complication of sickle cell anemia that occurs in 5% to 10% of adult patients with SCD. With good localized treatment, many ulcers heal within a few months. Leg ulcers that persist beyond 6 months may require skin grafting, although results of this treatment have been disappointing. Recent advances in management include the use of platelet-derived growth factor, prepared either autologously (Procuren) or by recombinant technology (Regranex).
234
569. Management of painful vasoocclusive crises in SCD patients frequently employs supplemental oxygen. Which of the following is (are) true about the supplemental oxygen administration in SCD patients? (1) Supplemental low-flow oxygen is often given to patients with SCD painful crisis in efforts to diminish the number of reversibly sickled cells (2) There is little supportive data for the use of supplemental oxygen in SCD patients (3) Routine oxygen administration in the absence of hypoxemia may impair reticulocytosis in SCD patients (4) Routine oxygen administration in the absence of hypoxemia has no proven benefit in SCD patients
569. (E)
235
570. Which of the following statement(s) is (are) true about epidemiology of SCD? (1) It is the most common hemoglobinopathy in the United States (2) The prevalence is significantly higher in the African American population than in the general population (3) It occurs in 0.3% to 1.3% of the African American population (4) The prevalence of SCD does not depend on the ethnic background of the population
.570. (A) Sickle cell anemia affects millions throughout the world. It is particularly common among people whose ancestors come from sub- Saharan Africa; Spanish-speaking regions (South America, Cuba, Central America); Saudi Arabia; India; and Mediterranean countries such as Turkey, Greece, and Italy
236
``` 571. Which of the following is (are) the measure(s) used to treat vasoocclusive crises of SCD during pregnancy? (1) Aggressive hydration (2) Supplemental oxygen in patients with hypoxemia (3) Partial exchange transfusions (4) Prophylactic transfusions ```
.571. (E) General management of vasoocclusive crisis during pregnancy begins with aggressive hydration to increase intravascular volume and decrease blood viscosity. Supplemental oxygen is essential in those patients with hypoxemia. Partial exchange transfusions are used to reduce polymerized hemoglobin S. Prophylactic transfusions may reduce the incidence of severe sickling complications during pregnancy.
237
572. When an opioid-tolerant patient in a sickle cell vasoocclusive crisis is admitted to a hospital, which of the following step(s) should be taken? (1) A baseline opioid infusion should be started immediately at an equianalgesic dose to patient’s home opioid requirement (2) A baseline infusion should be supplemented with a patient-controlled analgesia (PCA) on demand for breakthrough pain (3) As patient’s new opioid requirement becomes known from the PCA history, conversion to a combination of longand immediate-release opioid can be undertaken (4) Fast opioid dose increases may lead to hypoxemia and/or hypercarbia, which may exacerbate sickling of erythrocytes
572. (E) Some patients with SCD are opioid-tolerant secondary to their home opioid management. Therefore, the home opioid requirement should be taken into account for faster and more efficient pain control of sickle crisis pain. Opioid titration, however, may require some additional care because hypoxemia and hypercarbia further exacerbate sickling of erythrocytes.
238
573. Autonomic dysreflexia (1) is usually triggered by a spontaneous sympathetic discharge above the SCI level (2) is rarely associated with headache (3) is never life-threatening (4) manifests itself with increased blood pressure
573. (D) Autonomic dysreflexia is a potential lifethreatening condition, which is triggered by sensory input below the lesion and manifests itself with increased blood pressure, headache, and a risk of cerebral hemorrhage and seizure.
239
574. Which of the following is (are) true about the anterior cord syndrome? (1) It is characterized by complete sensory loss (2) Prognosis for motor function recovery is very poor (3) It is a complete SCI syndrome (4) It is characterized by complete motor function loss
574. (C) Anterior cord syndrome is a common incomplete cord syndrome. A patient with anterior cord syndrome may exhibit complete motor and incomplete sensory loss, with the exception of retained trunk and lower extremity deep pressure sensation and proprioception. This syndrome carries the worst prognosis for return of function, and only a 10% chance of functional motor recovery has been reported.
240
575. Which of the following is (are) true about the posterior cord syndrome? (1) It is characterized by preservation of temperature sensation (2) It is characterized by preservation of normal gait (3) It is uncommon (4) It is characterized by preservation of proprioception
575. (B) Posterior cord syndrome is a rare incomplete cord syndrome consisting of loss of the sensations of deep pressure and deep pain and proprioception, with otherwise normal cord function. The patient ambulates with a footslapping gait similar to that of someone afflicted with tabes dorsalis.
241
``` 576. Which of the following is (are) true feature(s) of Brown-Séquard SCI syndrome? (1) Ipsilateral motor deficit (2) Contralateral pain sensation deficit (3) Contralateral temperature sensation deficit (4) Uncommonness ```
576. (E) Brown-Séquard syndrome is an uncommon incomplete spinal cord syndrome. It is anatomically a unilateral cord injury, such as a missile injury. It is clinically characterized by a motor deficit ipsilateral to the SCI in combination with contralateral pain and temperature hypesthesia. Almost all these patients show partial recovery, and most regain bowel and bladder function and the ability to ambulate.
242
577. Anticonvulsants are commonly used for the treatment of neuropathic pain in the SCI patients. Which of the following correctly describe(s) their pharmacologic actions? (1) Modulation of calcium channels (2) Modulation of sodium channels (3) Increase of GABA inhibition (4) Blockade of reuptake of norepinephrine
577. (A) Anticonvulsants have several pharmacologic actions, such as modulation of sodium and calcium channels, increasing GABA inhibition, and suppressing abnormal neuronal hyperexcitability, which suggest an effect in neuropathic pain
243
578. Which of the following is (are) the usual symptom( s) of the autonomic dysreflexia? (1) Dramatic rise in blood pressure (2) Flushing and sweating in areas above the SCI (3) Marked reduction in peripheral blood flow (4) Decline in heart rate
578. (E) Patient with autonomic dysreflexia usually exhibit decline in heart rate, dramatic changes in blood pressure, flushing and sweating above the level of the injury, and a marked reduction on peripheral blood flow through the reflex pathways in the preserved vagus nerves.
244
579. Heterotopic ossification (HO) is commonly seen in patients with traumatic brain injury (TBI), cerebral vascular accident, burns, trauma, total joint arthroplasty, and SCI. Which of the following is (are) true about HO in SCI patients? (1) It is always painful (2) Hip is the most commonly affected (3) It is defined as ossification inside the joint capsule (4) Osteoclast inhibitors are use for both treatment and prophylaxis of HO
579. (C) HO is the formation of mature, lamellar bone in nonskeletal tissue, usually occurring in soft tissue surrounding joints. The bone formation in HO differs from other disorders of calcium deposition in that HO results in encapsulated bone between muscle planes, not intraarticular or connected to periosteum. In neurogenic HO secondary to TBI or SCI, the hip is the most common joint affected. Even though the most common symptom of HO is pain, it may be painless in patients with complete SCI. Etidronate disodium, a bisphosphonate, is an osteoclast inhibitor. It is structurally similar to inorganic pyrophosphate and is shown to delay the aggregation of apatite crystals into large, calcified clusters in patients with TBI and SCI. The recommended prophylactic treatment for HO in SCI is 20 mg/kg/d for 2 weeks, then 10 mg/kg/d for 10 weeks. The current treatment recommendation for established HO is 300 mg IV daily for 3 days followed by 20 mg/kg/d for 6 months in spinal cord patients.
245
``` 580. CRPS in the initial stages may be associated with (1) neurogenic inflammation (2) higher local levels of tumor necrosis factor alpha (3) high systemic CGRP levels (4) Increase in protein concentration in fluid of affected joints ```
580. (E) Localized neurogenic inflammation may explain the acute edema, vasodilation, and sweating observed in early stages of CRPS. Increased protein concentration and synovial hypervascularization is observed in the intraarticular fluid of affected joints. Findings that support the role of neurogenic inflammation in the generation of CRPS include elevated systemic levels of CGRP and local increase of IL-6 and tumor necrosis factor alpha in artificially produced blisters.
246
581. Which of the following is true about motor abnormalities in CRPS? (1) Dystonia of the hand or affected foot occurs in about 30% of the patients in the acute stages (2) Decrease active range of motion and increase amplitude of physiological tremor is seen in about 50% of the patients (3) They are likely related to an abnormal peripheral process (4) They may be explained by abnormalities in the cerebral motor processing
581. (C) Up to 50% of CRPS patients show decrease range of motion, increase amplitude of physiologic tremor, and reduce active motor force, with dystonia of the affected limb observed in only 10% of the chronic cases. Those motor changes are unlikely associated with a peripheral process and more likely the result of changes of activity in the motor neurons which point to abnormalities of cerebral motor processing.
247
``` 582. In terms of CRPS which of the following is (are) true? (1) Incidence of CRPS is 20% after brain lesion (2) Affected extremities after brain injury are at higher risk of developing CRPS than unaffected (3) CRPS following SCI is frequent (4) Upper extremities are more commonly affected than lower extremities ```
582. (C) It is estimated that the risk of CRPS after fractures is 1% to 2% and 12% after brain lesions. Retrospective studies in large cohorts shows a distribution in the upper and lower extremity from 1:1 to 2:1. CRPS following SCI are rare. Affected extremities after brain injury are more likely affected than unaffected ones.
248
583. Which of the following is true regarding bone scintigraphy? (1) The three stages of the three-phase bone scan include the perfusion, blood-pool, and mineralization phases (2) Homogeneous unilateral hyperperfusion in the perfusion phase is consistent with CRPS (3) Homogeneous unilateral hyperperfusion in the blood-pool phase is consistent with CRPS (4) Patients with CRPS show increase unilateral periarticular trace uptake in the mineralization phase
583. (E) The three stages of the three-phase bone scan include the perfusion phase 30 seconds postinjection, the blood-pool is 2 minutes postinjection, and mineralization phases is evaluated 3 hours postinjection. Homogeneous unilateral hyperperfusion in the perfusion and blood-pool phase is consistent with CRPS and excludes the differential diagnosis of osteoporosis because of inactivity. The mineralization phase in patients with CRPS shows elevated unilateral periarticular uptake.
249
``` 584. Which of the following is true regarding C fiber impulses? (1) After sensitization, antidromic impulses to peripheral C fiber terminals release vasoactive substance (2) Neurally released substances trigger neurogenic inflammation (3) Neurogenic inflammation includes axonal reflex, vasodilation, and plasma extravasation (4) C fiber activation peripherally releases CGRP and substance P ```
584. (E)
250
``` 585. The major peripheral pathologic finding(s) in patients with CRPS is (are) (1) patch atrophy of some muscle cells (2) capillary microangiopathy (3) Wallerian degeneration (4) generalized osteopenia ```
585. (A) The major peripheral pathologic findings in CRPS patients include (a) patchy atrophy of some muscle cells, secondary to disuse and nerve damage; (b) capillary microangiopathy, with accelerated turnover of endothelial cells and pericytes; (c) Wallerian degeneration of several types of axons; and (d) focal osteopenia in the territory innervated by a damaged nerve, and synovial cell disorganization and edema.
251
``` 586. Which of the following is (are) true regarding CRPS? (1) Medical procedures are the second most common cause of CRPS (2) Decrease deep tendon reflexes are a result of muscle atrophy (3) Cutaneous dynamic mechanical allodynia is a hallmark of central sensitization (4) Hypoesthesia may be rarely seen in patients with CRPS ```
586. (B) Medical procedures are the second most common cause of CRPS. The finding of exaggerated deep tendon reflexes in CRPS patients has been attributed to cortical disinhibition. Focal deficit of touch (hypoesthesia) were present in 50% of patients. Brushing skin activates low threshold mechanoreceptors which under normal circumstances has no connections with central pain neurons. Brush evoked pain (cutaneous dynamic mechanical allodynia)is a hallmark of central sensitization.
252
587. Characteristics of CRPS I in pediatrics include (1) CRPS I is more common in girls (2) the lower extremity is more often affected (3) CRPS may have genetic predisposition (4) CRPS is more common in Hispanics
587. (A) In contrast to CRPS II which has similar frequencies in boys and girls, CRPS I is more common in girls with a ratio of 4:1. The lower extremity is more affected (5:1 ratio). CRPS I is more common in Caucasians. There is also evidence that CRPS may have a genetic predisposition, with increase incidence in patients with HLAA3, B7, and DR2.
253
588. Characteristics of CRPS II in pediatrics include (1) the incidence is similar in boys and girls (2) brachial plexus injury during delivery commonly leads to chronic pain (3) Erb palsy do not generally develop CRPS (4) patients with Erb palsy need a comprehensive treatment to avoid the development of CRPS II
588. (B) The gender distribution of CRPS II in the pediatric population is roughly similar in boys and girls. Even though patients with brachial plexus injury during delivery (Erb palsy) is common and can lead to prolonged motor weakness, they rarely develop pain. Interestingly most of these patients do better without treatment.
254
589. To confirm the diagnosis of CRPS: (1) There are no laboratory tests to confirm the diagnosis (2) Disturbed vascular scintigraphy is necessary to make the diagnosis of CRPS (3) Bone scan is nonspecific for the diagnosis of CRPS (4) There is no utility in ordering bone scan in patients with CRPS
589. (B) The diagnosis of CRPS remains a clinical decision based on findings in the history and physical examination. There are no laboratory tests that can absolutely confirm or exclude the diagnosis. Although controversial, most of the authors find that bone scans are quite nonspecific for the diagnosis of CRPS. Patients with a clinical diagnosis of CRPS may have bone scans showing hypofixation or hyperfixation or may be normal. The primary utility of the bone scan could be in ruling out some underlying orthopedic abnormality that might be triggering neurovascular changes that may confused the findings with those of CRPS.
255
590. Which of the following is true regarding movement disorders in CRPS patients? (1) Motor dysfunction is the result of voluntary defensive response to protect the limb from painful stimuli (2) Deep tendon reflexes in these patients are normal to brisk (3) Movement disorders often happen in early stages of the disease (4) Akinesia is a prominent finding in CRPS patients
590. (C) Movement disorders are an essential feature of patients with CRPS. Motor dysfunction is not simply a voluntary defensive response to protect the limb from painful stimuli, but may represent the interaction of peripheral and central mechanisms. Deep tendon reflexes are often brisk. The prevalence of movement disorders increases with the duration of the disease. One characteristic form of these movement disorders is the presence of inability to start a movement (akinesia).
256
591. In terms of CRPS and dystonia, which is characterized by involuntary contractions of one or more muscles, it can be said that (1) dystonia is a prominent feature of CRPS (2) dystonia in patients with CRPS typically presents with flexure postures (3) tonic dystonia often spares the first two digits (4) extensor postures occur early in the development of dystonia
591. (A) Dystonia in CRPS patients causes twisting movements or abnormal postures of the affected body parts. The most prominent motor feature is flexor postures (tonic dystonia) of the fingers, feet, and wrist. Extensor postures occur but are rare.
257
592. Supraspinal regulatory mechanisms that may explain some of the features of CRPS include (1) spread of cortical representation of the affected limb (2) patients with generalized dystonia have increased intracortical excitability to sensory stimuli (3) motor cortical disinhibition (4) early increase activity of the thalamus contralateral to the affected limb
592. (E) Supraspinal mechanisms play a major role in the abnormal sensory perception of patients with CRPS. Segmental dystonia is characterized by spread of the cortical representation of the affected extremity and its corresponding synaptic connections to adjacent cortical areas. On the other hand, generalized dystonia have increased intracortical excitability to sensory stimuli, and motor cortex disinhibition has been confirmed in CRPS I. PET scan and SPET have shown increased activity of the contralateral thalamus in patients in early stages of CRPS and hypoperfusion in advanced stages.
258
``` 593. Which of the following is (are) myofascial trigger point characteristic(s)? (1) Weakness with muscle atrophy (2) Referral of pain to a distant site upon activation of the trigger point (3) Range of motion not restricted (4) Autonomic phenomenon, such as piloerection or changes in local circulation (regional blood flow and limb temperature) in response to trigger point activation ```
593. (C) Myofascial trigger point characteristics: • Focal severe tenderness in a taut band of muscle • Referral of pain to a distant site upon activation of the trigger point • Contraction of the taut band (local twitch response) upon mechanical activation of the trigger point • Reproduction of the pain by mechanical activation of trigger point • Restriction of range of motion • Weakness without muscle atrophy • Autonomic phenomenon such as piloerection or changes in local circulation (regional blood flow and limb temperature) in response to trigger point activation Individual features of the trigger point are differentially represented in different muscles. An examiner should not expect to find each feature of the trigger point in every muscle by physical examination.
259
594. The definitive goal of treatment of persons with myofascial pain syndrome is (are) (1) restoration of function through inactivation of the trigger point (2) restoration of normal tissue mobility (3) relief of pain (4) increased range of motion
594. (A) Inactivation of the trigger point is a means to achieve pain relief, to improve biomechanical function, and then to improve the ability of the patient to better perform whatever tasks have been selected as goals. Relief (not elimination) of pain or increased range of motion, both of which can be the result of trigger point inactivation, are not in themselves the goals of treatment.
260
595. Inactivation of the myofascial trigger point can be accomplished (1) manually (2) by direct injection of a local anesthetic into the muscle (3) by dry needle intramuscular stimulation of the myofascial trigger point (4) by correcting structural mechanical stressors
595. (E) While chronic myofascial pain syndrome is best treated with a multidisciplinary team approach including the patient, physicians, psychologists, clinical social workers, occupational therapists, physical therapists, ergonomists, massage therapists, and others actively involved in patient care; patients with acute myofascial pain syndrome may only require treatment by physicians and physical therapists. Too frequently, patients with chronic myofascial pain are started too soon on isotonic training and conditioning, causing further aggravation of active trigger points and an increase in pain and dysfunction. The acute treatment plan may be divided into a pain-control phase and a training or conditioning phase. During the pain-control phase, the most essential component is inactivation of the trigger point. Patients must change their behaviors and avoid overstressing their muscles without becoming excessively inactive. The pain-control phase must have a definitive endpoint. If patients do not move beyond the paincontrol phase to the conditioning phase, patients can be restricted in their functional abilities and be at greater risk of reinjury. The training or conditioning phase follows and it involves therapeutic exercises, movement reeducation, and overall conditioning
261
``` 596. With regard to trigger point injections, botulinum toxin which of the following is true? (1) Has been tried unsuccessfully in myofascial trigger point inactivation (2) Can cause a flulike myalgia (3) Occasionally causes weakness that is confined to the area of injection (4) Is a long-lasting trigger point injection capable of about a 3 month inactivation of the trigger point ```
596. (C) Botulinum toxin has been tried successfully in myofascial trigger point inactivation; however, it can cause a flu-like myalgia that lasts days to a week and sporadically causes weakness beyond the area of injection. It functions as a long-lasting trigger point injection that can provide up to 3 months of relief in contrast to the days to 1-week effect of traditional trigger point injection with local or no anesthetic.
262
597. A 40-year-old female with chronic myofascial neck pain wants to go to an acupuncturist. She should know that (1) in one study it was found that shallow needling reduced the pain of chronic myofascial neck pain (2) in a randomized, double-blind, shamcontrolled study, acupuncture was found to be superior to dry needling in improving range of motion (3) in a randomized, double-blind, shamcontrolled study, acupuncture was found to be better than placebo when treating trigger points in chronic neck pain (4) in a randomized, double-blind, controlled study, acupuncture was found to be better than control only when it was followed by transcutaneous electrical stimulation
597. (A) 1. Japanese acupuncture or shallow needling reduced the pain of chronic myofascial neck pain in one study. 2. and 3. These have been found to be true. 4. While this study does not exist, a technique of dry needling called intramuscular stimulation does exist. It involves the insertion of the needle into the taut band without necessarily considering the actual trigger point. It may be combined with electrical stimulation delivered through the needle (percutaneous electroneutral stimulation).
263
598. A 40-year-old woman comes to the pain clinic for initial evaluation. After a thorough history and physical examination, the patient is diagnosed with fibromyalgia. Which symptom(s) would support your diagnosis of fibromyalgia as opposed to myofascial pain syndrome? (1) Widespread pain (2) Irritable bowel syndrome (3) Distal paresthesias (4) Occipital headaches
598. (E)
264
599. Which of the following is (are) true of fibromyalgia? (1) Adult women are twice as likely to be affected as adult men (2) Prevalence peaks in the fourth decade of life (3) Many children diagnosed with fibromyalgia will have worsening of their symptoms as they reach adulthood (4) Affects all ethnic groups
599. (D) Fibromyalgia has been found among all cultures throughout the world with an incidence of 2% to 12% of the population. In adulthood, women are affected four to seven times as often as men. The frequency of fibromyalgia increases with age and peaks in the seventh decade of life. In childhood, boys and girls are affected equally. In contrast to adults, children’s symptoms may resolve with age.
265
``` 600. Risk factors for the development of fibromyalgia syndrome include (1) physical trauma (2) febrile illness (3) family history of fibromyalgia (4) history of sexual abuse ```
600. (A) Although research is ongoing, the development of fibromyalgia appears to be increased if the patient has had a febrile illness, a history of physical trauma, or a family history of fibromyalgia syndrome. Approximately onethird of patients with fibromyalgia report that another member of their family has previously been diagnosed with fibromyalgia
266
601. Sleep disturbances are common in patients with fibromyalgia. Difficulties the patient may encounter include (1) problems initiating sleep (2) awakening in the middle of the night (3) light, unrefreshing sleep (4) difficulty napping throughout the day
601. (E) Greater than 90% of patients with fibromyalgia suffer from chronic insomnia. Some patients may have problems falling asleep. Other may awaken a few hours after going to sleep and feel alert, thus disrupting their sleep throughout the remainder of the night. After a night of sleep, patients with fibromyalgia may feel stiff, tired, and “cognitively sluggish.” These patients also encounter difficulty napping throughout the day. Patients with fibromyalgia have disrupted sleep architecture with alpha wave intrusions in deep, delta wave sleep.
267
602. Pathophysiologically, fibromyalgia (1) is a disorder of abnormal processing of sensory information within the CNS (2) exhibits a narrow array of recognized objective physiological and biologic abnormalities (3) patients demonstrate abnormally low regional cerebral blood flow in thalamic nuclei and other pain-processing brain structures that is inversely correlated with spinal fluid substance P levels (4) demonstrates abnormal spinal cord windup
602. (E) 3. In fibromyalgia patients, CT has revealed unusually low cerebral blood flow in the thalamic nuclei, the left and right heads of the caudate nucleus, and the cortex that correlates with spinal fluid substance P levels. 4. Windup is a frequency-dependent increase in the excitability of spinal cord neurons, evoked by electrical stimulation of afferent C fibers. Glutamate (NMDA) and tachykinin NK1 receptors are required to generate windup and therefore a positive modulation between these two receptor types has been suggested. Whatever the mechanisms involved in its generation, windup has been interpreted as a system for the amplification in the spinal cord of the nociceptive message that arrives from peripheral nociceptors connected to C fibers. This probably reflects the physiological system activated in the spinal cord after an intense or persistent barrage of afferent nociceptive impulses. On the other hand, windup, central sensitization and hyperalgesia are not the same phenomena, although they may share common properties. Spinal cord windup is abnormal in fibromyalgia syndrome. Physical trauma or a fever/infection may be provisionally related to the onset of fibromyalgia in over 60% of cases.
268
603. That same patient (who happens to be a neurobiology graduate student) starts to ask about the role that cytokines play in fibromyalgia. You tell her that (1) IL-8 has been found to be significantly higher in the serum of fibromyalgia patients, especially in depressed patients (2) IL-6 was not found to be increased in the blood of fibromyalgia patients (3) the production of IL-8 in vitro is stimulated by substance P (4) cytokines do not play a role in the pathogenesis of fibromyalgia
603. (B) The levels of serum IL-8 were higher in fibromyalgia patients, and IL-6 was statistically higher in cultures of fibromyalgia peripheral blood mononuclear cells compared with in controls. The IL-8 increase was most dramatic in depressed patients, but there was also a correlation with the duration of fibromyalgia and the pain intensity. The production of IL-8 in vitro is stimulated by substance P.
269
604. Which of the following medications are FDA approved for the treatment of fibromyalgia? (1) Cyclobenzaprine (2) Duloxetine (3) Tramadol (4) Pregabalin
604. (D) Pregabalin has the potential to raise the threshold for pain fiber depolarization. It is a ligand for the α2δ subunit of voltage-dependent calcium channel receptors, which has analgesic, anxiolytic-like, and anticonvulsant activity. It decreases the release of numerous neuropeptides, including noradrenaline, glutamate, and substance P. It has already been approved for treating partial seizures, pain following the rash of shingles and pain associated with diabetes nerve damage (diabetic neuropathy). Two double-blind, controlled clinical trials, involving about 1800 patients, support approval for use in treating fibromyalgia with doses of 300 or 450 mg/d. It is effective in reducing the severity of body pain, improving quality of sleep, and reducing fatigue in fibromyalgia patients. Pregabalin was approved by the FDA for use in fibromyalgia patients on June 21, 2007. The most common side effects of pregabalin include mild to moderate dizziness and sleepiness. Blurred vision, weight gain, dry mouth, and swelling of the hands and feet also were reported in clinical trials. The side effects appeared to be dose-related. Pregabalin can impair motor function and cause problems with concentration and attention. The FDA advises that patients talk to their doctor or other health care professional about whether use of pregabalin may impair their ability to drive. 1. Cyclobenzaprine, have been used in the past with success in fibromyalgia patients, they have not been approved by the FDA. 2. Duloxetine is an FDA-approved treatment for major depression, neuropathic pain from diabetic peripheral neuropathy, and generalized anxiety disorder. The drug is a serotonin and norepinephrine reuptake inhibitor that exhibits nearly equal serotonin and noradrenaline reuptake inhibition. Atrial of duloxetine for patients with chronic pain and/or major depression indicated that for the fibromyalgia patients, 80% of the observed effect on pain is a direct analgesic effect rather than an indirect antidepressant effect. Common adverse events were: nausea, headache, dry mouth, insomnia, constipation, dizziness, fatigue, somnolence, diarrhea, and hyperhidrosis. Two placebo-controlled randomized studies on the treatment of fibromyalgia-associated pain with duloxetine have been published. Both studies demonstrated that duloxetine treatment improved fibromyalgia-associated pain in women. However, the medication has not yet been approved for the treatment of fibromyalgia. Another type of serotonin and noradrenaline reuptake inhibitor is represented by milnacipran, where noradrenaline reuptake inhibition is favored over that of serotonin. Some reports state that milnacipran may also be effective in treating fibromyalgia body pain. 3. Tramadol has only recently been shown to improve the pain of patients with fibromyalgia. It combines weak μ-agonist with NMDA antagonist and noradrenaline and serotonin reuptake inhibition. In the combination preparation that comes with acetaminophen, a considerable synergy has been noticed. Nausea and dizziness can be limiting at first in about 20% of patients, but starting with just one tablet at bedtime for 1 to 2 weeks can decrease the prevalence and allow later increases but about one tablet every 4 days to full therapeutic levels. Atypical tramadol regimen for fibromyalgia is 300 to 400 mg/d in three or four divided dosages, concomitant with acetaminophen at 2 to 3 g/d in divided doses. Administering 5-hydroxytryptophan can augment the synthesis of serotonin. Onehundred milligrams, orally, three times daily has been shown to be an effective dose in treating fibromyalgia.
270
605. The pain in fibromyalgia is, at least in part, mediated by central sensitization. Studies have shown that (1) dextromethorphan and ketamine may improve pain and allodynia in fibromyalgia patients (2) the majority of patients with fibromyalgia that tried ketamine benefited (3) ketamine’s efficacy was limited because of its side effects (4) dextromethorphan had a similar sideeffect profile
605. (B) Central sensitization can be inhibited by NMDAreceptor blockade. Two NMDA receptor antagonists, ketamine and dextromethorphan (an oral preparation) have been found to exhibit favorable effects on pain and allodynia in fibromyalgia patients. With ketamine, 50% of patients benefited. Fibromyalgia subgroups of responders and nonresponders were perpetuated by these findings because all the fibromyalgia patients in the study were otherwise comparable. Ketamine’s effectiveness was limited because of its frequently occurring psychotropic side effects, such as feelings of unreality, altered body image perception, aggression, anxiety, nausea, dizziness, and modulation of hearing and vision. Dextromethorphan has a better side-effect profile than ketamine. It was administered with tramadol to increase the antinociceptive effect, to hold adverse effects low, and to decrease the development of opioid tolerance. A good response was obtained in 58% of the fibromyalgia patients who tried this regimen. It may be a consideration for the patients who respond positively to IV ketamine.